Этого треда уже нет.
Это копия, сохраненная 18 июня 2019 года.

Скачать тред: только с превью, с превью и прикрепленными файлами.
Второй вариант может долго скачиваться. Файлы будут только в живых или недавно утонувших тредах. Подробнее

Если вам полезен архив М.Двача, пожертвуйте на оплату сервера.
2 461535
Если фотон-квант электромагнитного поля, то почему магнит магнитит, а фонарик не магнитит??
3 461537
>>61535
А, мне уже вроде более-менее как-то ответили на это в прошлом треде.

Тогда перекат другого вопроса оттуда, тесно граничащего с этим вопросом и ответами:

Если это существует/происходит, то это можно описать именно так, как оно реально происходит, а не абстрактно.
Да/нет?
4 461538
>>61537
Определи значение термов "Существует", "Происходит", "Реально происходит", "Абстрактно просходит"
5 461539
>>61535
Потому что элктро-магнитное поле. Чтобы было магнитное поле, нужно наличие электрического поля. Электрическое поле создается только заряженными частицами.
См. уравнения Максвелла.
6 461544
>>61538
Это оказалось сложнее чем я думал.

Посему быстренько переобуюсь и спрошу:
Так всё-таки поля, или частицы? Кванты полей реально есть, или это всё что-то типа dv, позволяющая делать разные дэвэшные по дэтэшные штучки?
Какие есть доказательства того что поля квантуются?
7 461548
>>61544

>Так всё-таки поля, или частицы?


Гладиолус? Не вижу тут никакого вопроса.

>Кванты полей реально есть, или это всё что-то типа dv, позволяющая делать разные дэвэшные по дэтэшные штучки?


Бессвязный набор слов.

>Какие есть доказательства того что поля квантуются?


Для описания физики микромира пришлось отказаться от алгебры функций с поточечным умножением и заменить ее на алгебру операторов.
8 461549
>>461229

>Там просто 5% понимают матан и кванты. Остальные 95% это просто уебки, насмотревшиеся научпопа.


Погуглил одну пасту и наткнулся на тред годовой давности в матаче о философии математики. Очень сочный тред.
Так забавно когда видишь там ту же хуиту что год спустя. Вроде почти дословных - в статфизике нет математики и еще всякое.
Особенно поучительно посмотреть на конструктивного петуха. Ему приводят железобетонный факт - абсолютному большинству практикующих математиков глубокой похуй на основания и уж тем более на его маня-интуиционизм/конструктивизм. На это конструктух всегда делает один и тот же ход - начинает верещать - да вы же даже не понимаете швятой изоморфизм Карри-Ховарда. Хотя ну блядь ну при чем тут он. И тем более делов там на пять минут можно объяснить.
Так же и ЦПТ-петухом. И еще можно набрать десятки примеров этого паттерна поведения.
9 461551
препод задал какую-то странную хуйню и просить поискать по ней инфы. мне кажется, он поехал. якобы он работает над следующей теорией:
электропроводимость дистиллированной воды зависит от дня недели.
звучало это именно так. типа сегодня проводимость одна, завтра -- другая. чзх?
10 461552
>>61549

> в матаче о философии математики.


Собственно поэтому матанач и пребывает в скверном состоянии. Конструктопетухи засирали любые треды.

> в статфизике нет математики


смеялись всей кафедрой.
А ведь еще был N-петух, с ним даже пикчи были.
11 461562
>>61548

>Гладиолус? Не вижу тут никакого вопроса.


>Бессвязный набор слов.


Тебе так только кажется.

>Для описания физики микромира пришлось отказаться от алгебры функций с поточечным умножением и заменить ее на алгебру операторов.


Вот это уже интересно, но вопрос пока не в этом.

>>61538
>>61537
Кстати вспомнил пример физической абстракции-электрон крутится по орбите вокруг ядра.
12 461563
>>61539

>Электрическое поле создается только заряженными частицами.


Электронами что ли? Они же вроде просто побочный вредный продукт.(в контексте передачи и использования электрической энергии)
13 461569
это типа получение раст при равноуск движ разными техниками

где 50м ?
Безымянный.png37 Кб, 1774x1179
14 461576
Будет ли за счет испарения(не обязательно воды, этанола например например) естественная циркуляция?
15 461589
вот смотри: нейтрино почти не взаимодействует с веществом (потому что слишком маленькие?); мы почти не взаимодействуем с тёмной материей. Может, мы просто в том же к ней отношении? Например, "атомы" тёмной тёмной материи размером с голактику, и
16 461613
>>61576
Гугли кольцар Лазарева, например.
17 461620
Возможно ли вспомнить каждый день своей жизни, например, с 5 до 15 лет, или почти всё безвозвратно забыто, а то и вообще не запоминалось?
18 461621
Во сколько раз умнее будет мегацефал, у которого в 2 раза больше нейронов в мозгу и вся энергоподводящая инфраструктура тоже?
19 461622
Зачем организму нужен сон для восстановления мышц, тканей и тд? Чё он так не может?
20 461624
>>61622
Сон нужен для перезапуска нервной системы, без сна обучение и память работали бы на порядок медленнее.
21 461625
>>61576
Воткни шаровой клапан и не еби мозги себе.
22 461634
>>61624
Это и без тебя знаю. Перечитай вопрос. Почему не может восстановить мышцы без сна?
23 461636
>>61634
Может. Утомленность =/= мышечная усталость.
24 461638
Что будет, если заменить днк на энантиомер во всех клетках? Ну или поменять спирали местами?
25 461641
>>61535

>то почему магнит магнитит, а фонарик не магнитит??


Магнетизм - одна из форм взаимодействия объектов, обладающих электрическим зарядом, у фотона нет заряда => профит. Но прикол - фотон взаимодействует с магнитным полем.
26 461644
Понимаю что вопросы могут быть тяжкими, но очень интересно:
1) Что именно происходит во время слияния антиматерии и материи? Ну, понятно что аннигиляция, выделение энергии, но какого-хуя? Можете это разъяснить, как разъяснили бы химическую реакцию?

2) От чего зависит, сколько молекул воды входит в кристаллогидрат? Т.е. я знаю Медный купорос - CuSO4*5H2O, почему не на 7h2o, как у железного купороса?
27 461651
>>61644

>но какого-хуя?


Простая модель: каждая частица, как колебание некоего поля, может быть двух видов, при сложении эти колебания обнуляются, а их энергия переходит в возмущение другого поля - безмассовые гамма кванты, так как просто взять и пропасть не может. Энергия просто кочует из одной формы в другую.
28 461655
Здравствуйте!
Почему трупы гниют? Почему здоровые живые существа не гниют, пока они живые?
Спасибо!
29 461664
>>61655
Потому что организм является относительно открытой системой, способной обмениваться веществом с окружающей средой, что возволяет выводить промежуточные продукты участвующие в гниении, т.е. централизованно и синхронно заботится о том, чтобы он не начал гнить, посредством клеточного метаболизма, гуморальной и нервной регуляции.
30 461682
Как там дела с наноассемблером? Всем похуй, все дрочат на Маска-Небоходца?
31 461684
>>61682

>Как там дела с наноассемблером?


Работает прямо у тебя внутри, прямо сейчас. Реплицирует ДНК. Уже миллион лет.
32 461685
>>61684
На нем нельзя из земли сделать железную пластину, нужен кастомный
33 461686
Может кто-нибудь просто и понятно объяснить как де всё-таки работают процессоры? Т.е. как работают вычисления на транзисторах.
Просто как по мне, транзистор это просто такая штука, которая или пропускае энергию, или не пропускает, в зависимости от того есть ли сигнал управляющий или нет.
Но мне тут вместо вычислений видется бесконечная рекурсия. Т.е. чтобы выислить(как часть вычисления) транзистор должен или пропустить ток, или нет. Но делает он это только в зависимости от того есть на нём сигнал или нет. Т.е. всё уже решено нашичием или отсутствием этого сигнала. А значит этот сигнал этому транзистору должен был подать другой транзистор, которому должен был падать сигнал другой транзистор, которому...
34 461687
>>61641
И как это объясняется? Или никак не объясняется?
35 461688
>>61686
Простейшую конструкцию из двух транзисторов можно поставить в два устойчивых состояния, создав таким образом ячейку памяти. Это триггер.
Два состояния можно перемножать и делать над ними логические операции.
А дальше ты просто хуяришь миллиарды вот етово вот и получаешь компьютер. Ничего сложного. Студент-электронщик на 2 курсе может сделать компьютер из глины и соломы, так, например, Эльбрус сделали.
36 461689
>>61688
Всё равно пока не понял.
Чтобы что-то с ними сделать, это опять-таки нужно подать сигнал...
oblivion.webm1,7 Мб, webm,
360x360, 0:14
37 461691
>>61689
Сигнал подаётся при загрузке. Можешь считать, что первый сигнал, который ты подаёшь пеке - нажатие на кнопку Power.
Потом с диска компульсивно грузится операционка, которая компульсивно выполняется на описанной логике. Ввод и вывод это считывание логических уровней из памяти. Это всё.

Перечни протоколов и сигналов содержатся в ОС и фирмваре. Это базовый уровень аппаратной абстракции. Потом идёт несколько уровней сверху, когда мы как бы забываем, что имеем дело с транзисторами в положении on/off. (Поэтому частоты компьютеров растут, а сами компьютеры эффективно быстрее не становятся. Мы просто докидываем уровень абстракции, чтобы меньше платить профессиональным кодерам и каждая домохозяйка могла кодить, но получается, как видишь, как всегда.)
38 461692
>>61691
Я не это спрашивал, я спрашивал именно про самый первый уровень. Про то как происходит вычислительние с помощью on/off положения транзистора. Элементарное вычисление, одно. Как происходит одно логическое действие с помощью одного транзистора или их какой-то связки.
40 461702
Как работает генная инженерия?
Видел как-то видос (или статью читал), в котором один генетик решил на себе исправить какую-то генную проблему. Там же упоминалось, что это очень больно. А как это происходит?
Первый интересующий меня вопрос: генная инженерия вообще как работает внутри организма?
И второе: как происходит ввод материала с генноизменяющей массой организм? Инъекции в спинной мозг? В головной мозг? В кровь? В мышцы?
41 461704
Как работает обратный агонизм?
42 461707
Какой самолетный двигатель более эффективный с точки зрения литров топлива на 1 км пути?
43 461710
>>61707
Пропеллер, естественно.
44 461711
>>61710
Почему тогда абсолютный рекорд дальности держит реактивный двигатель?
45 461725
1.Какая разница между:
- pn-переходом, который пропускает току только в одну сторону.
- просто контактом к полупроводнику или между ними, пропускающий линейно.
- банальным соединением двух полупроводников на воздухе руками(или тисками).
Почему в одном случае появляется pn-переход а в другом это просто контакт?

2.Возможно ли гальваническое разделение легирующих примесей в твердом или жидком полупроводнике? Если да, то возможно ли появление таким образом pn-перехода в готовом кристалле?

3. Как проявляется эффект Пельтье в pn-переходе и обычном контакте? Если диод включить в обратку, будет ли он охлаждаться? Если светодиод излучает свет в видимом диапазоне, значит ли это что контакт pn-перехода будет нагреваться меньше чем охлаждаться контакт металл-полупроводник?
46 461726
>>61711
Турбореактивный вообще-то. Это и есть пропеллер 2.0.
mikrovolnovka.jpeg58 Кб, 620x399
Микроскопический ядерный реактор 47 461727
Анон, такой вопрос. Возможно ли засунуть нестабильный изотоп в устройство, которое будет его нагревать и сжимать, чтобы снизить критическую массу ядерной реакции и получить тем самым серию микроскопических контролируемых ядерных взрывов?
mikrovolnovka - копия.jpeg219 Кб, 2972x1084
48 461729
Такую вот фигню можна запилить
49 461730
как работает планковская экономика?
50 461731
>>61727
Вероятность ядерных реакций практически НЕ зависит от термодинамического давления/температуры, ДО пороговых значений при которых собственно начинаются ТЕРМОядерные реакции. Самая "легко" осуществимая - это D + T (термоядерная бомба, токамаки, ИТЕР - вот это все).

>критическую массу


- так не изменить. Можно изменить критические условия - например уменьшив площадь плутониевого шара (что определяет вероятность утечки нейтронов с поверхности) - обжатием взрвчткой - это и делается в ядерных, плутониевых бомбах.
51 461732
>>61731

>обжатием взрвчткой


Значит можно обжимать не только взрывчаткой?

>Можно изменить критические условия


Ну вот это как вариант как то поменять.
52 461733
>>61730
Просто без задней мысли.
53 461735
>>61732

>Значит можно обжимать не только взрывчаткой?


Да, можно обжимать излучением (там на самом деле механизм сложнее), например от лазера - так работает
https://ru.wikipedia.org/wiki/Инерциальный_управляемый_термоядерный_синтез
или другого ядерного взрыва - так работает термоядерная бомба. Но это для синтеза, а не деления. И причем здесь "микровзрывы" урана - я так и не понял.
И ты похоже сути процесса взрыва делящихся материалов не понимаешь.
54 461736
>>61735

>Но это для синтеза, а не деления.


Да я то это понимаю, но мне подумалось, что нестабильный изотоп какой нибудь сильно склонный к делению можно так сказать подтолкнуть, запихав его в некий ананлог машины для синтеза, где его сжать и нагреть. Это необязательно должен быть уран, цезий например ил что там еще есть, чем нестабильнее тем лучше, как вариант вообще тритий.
55 461737
>>61736

>нестабильный изотоп какой нибудь сильно склонный к делению можно так сказать подтолкнуть


- Да, это так. Но для этого нужны "ядерные" энергии. (В принципе ансамблю нуклонов ядра можно даже приписать некую температуру и говорить "разогреве ядра".) Например это "легко" делается добавлением лишней частицы в ядро - но из-за кулоновсгого барьера, действительно легко добавить можно только нейтральную частицу - так работает реакция вынужденного деления от нейтрона - она и обеспечивает цепную реакцию в бомбах и реакторах.

>чем нестабильнее тем лучше, как вариант вообще тритий.


- еще раз, температуры должны быть соответствующие.

>Вероятность ядерных реакций практически НЕ зависит от термодинамического давления/температуры, ДО пороговых значений при которых собственно начинаются ТЕРМОядерные реакции.



И кстати, с чего ты взял, что повышение температуры обязательно увеличит вероятность распада? Бывает внешние условия, наоборот, стабилизируют частицу. Например в стабильных ядрах или нейтронных звездах - нейтроны стабильны, а свободный нейтрон распадается бета-распадом, с тау =~ 15 минут.
56 461738
>>61736

>к делению можно так сказать подтолкнуть


Кстати, пытались копать в другую сторону - наоборот - превратить нестабильный изотоп, в стабильный - "заморозив" его квантовым эффектом не закипающего чайника (квантовый эффект Зенона). Не вышло, ЕМНП.
57 461739
>>61725
1. pn-переход возникает из-за что, в одном целом полупроводнике, где могут существовать и электроны, и дырки(и еще много других носителей заряда), есть области с явным предпочтением разных носителей заряда (дырок/электронов). В итоге в области где контачаться две эти разные области, постоянно рекомбинируют носители заряда, тем самым "уничтожая" ток. Для предотвращение беспредела нужно подавить в какой либо области все дырки/электроны, чтоб восторжествовали только одни дырки/электроны. На этом же принципе работает транзистор.
Однако, в каждом материале есть только свои разрешенные носители заряда и на поверхности границ возникает некоторый барьер, на котором происходит преобразование носителей заряда(можно легально преобразовать дырки в электроны без рекомбинации). Но контакт металл-полупроводник(даже два полупроводника) вообще говоря не линейный. Может возникать ситуация, при токе часть энергии-импульса будет теряться, потому что разрешенные энергии носителей заряде в полупроводнике не соответствую металлу, так возникает эффект Шоттки, на котором основанны одноименные диоды.
В прицепе возможно соединить два куска полупроводника, чтоб они потом стали одним целом. Но на практике это практически не выполнимо. Все из-за того, что поверхность материала эта его экстремальная часть, на котором локализуются очень много эффектов. В первую очередь создается барьер по преобразованию носителей заряда, где дырки с электронами не рекомбинируют. И когда два куска полупроводника сжимаешь вместе их поверхностные эффекты на месте контакта только усиливаются, а не ослабляются. Получается все что угодно не один цельный полупроводник. Короче задача сводится с созданию одного цельного кристалла из двух, без повреждением кристаллической решетки.
2. Нахуй это надо? Суть pn-перехода в отсутствие поверхности разрыва, а любая гальваника их прямо таки плодит. Уже давно изобрели способы эпитаксии с запредельными характеристиками.
3. Для эффекта Пельте нужен ярко выраженная поверхность раздела. Т.е он не происходит на pn-переходе, а на границе полупроводника с другим материалом.
Нет. нет.
57 461739
>>61725
1. pn-переход возникает из-за что, в одном целом полупроводнике, где могут существовать и электроны, и дырки(и еще много других носителей заряда), есть области с явным предпочтением разных носителей заряда (дырок/электронов). В итоге в области где контачаться две эти разные области, постоянно рекомбинируют носители заряда, тем самым "уничтожая" ток. Для предотвращение беспредела нужно подавить в какой либо области все дырки/электроны, чтоб восторжествовали только одни дырки/электроны. На этом же принципе работает транзистор.
Однако, в каждом материале есть только свои разрешенные носители заряда и на поверхности границ возникает некоторый барьер, на котором происходит преобразование носителей заряда(можно легально преобразовать дырки в электроны без рекомбинации). Но контакт металл-полупроводник(даже два полупроводника) вообще говоря не линейный. Может возникать ситуация, при токе часть энергии-импульса будет теряться, потому что разрешенные энергии носителей заряде в полупроводнике не соответствую металлу, так возникает эффект Шоттки, на котором основанны одноименные диоды.
В прицепе возможно соединить два куска полупроводника, чтоб они потом стали одним целом. Но на практике это практически не выполнимо. Все из-за того, что поверхность материала эта его экстремальная часть, на котором локализуются очень много эффектов. В первую очередь создается барьер по преобразованию носителей заряда, где дырки с электронами не рекомбинируют. И когда два куска полупроводника сжимаешь вместе их поверхностные эффекты на месте контакта только усиливаются, а не ослабляются. Получается все что угодно не один цельный полупроводник. Короче задача сводится с созданию одного цельного кристалла из двух, без повреждением кристаллической решетки.
2. Нахуй это надо? Суть pn-перехода в отсутствие поверхности разрыва, а любая гальваника их прямо таки плодит. Уже давно изобрели способы эпитаксии с запредельными характеристиками.
3. Для эффекта Пельте нужен ярко выраженная поверхность раздела. Т.е он не происходит на pn-переходе, а на границе полупроводника с другим материалом.
Нет. нет.
58 461740
>>61737

>И кстати, с чего ты взял, что повышение температуры обязательно увеличит вероятность распада


Я имел ввиду одновременное повышение температуры и давления естественно.
В целом моя идея в том чтобы упаковать большое количество энергии в маленькой массе и объеме. Меня в последнее время вообще весьма интересует это направление. Читал что в некий гипотетический двигатель вообще можно заливать раствор солей урана, где они будут делиться при каких то не помню каких условий, возможно температура плюс давление плюс облучение какое нибудь тоже крайне плотное.
38.jpg77 Кб, 500x476
59 461741
>>61740
Ты заебал.

>Читал что в некий гипотетический двигатель вообще можно заливать раствор солей урана, где они будут делиться при каких то не помню каких условий


Да, такие двигатели есть и они, блядь, НЕ "гипотетические" - возим стакан соли плутония и он вскипит и брызнет тебе в тупую рожу.
Но, нет - никакие реалистичные температуры/давления на вероятность реакции в таком двигателе не влияют.

>плюс облучение какое нибудь


- Да, блядь, импульсный нейтронный инициатор (тогда с полонием, а сейчас с миниатюрным ускорителем) используют в бомбах с первых лет. Но это не для снижения крит-массы, а для надежного и предсказуемого начала цепной реакции, т.к для него (такого ИНИ) тоже нужна энергия.
60 461742
>>61741

>Я имел ввиду одновременное повышение температуры и давления естественно.


Еще раз - с какого ты решил, что это хоть как-то повлияет на элемент "N"? (Если речь не о условиях в центре сверхновых звезд при которых этот "N" и образовался.)
61 461743
Это
>>61742
сюда
>>61740
62 461744
>>61740
Чтобы замедлить распад, ядро как систему нужно запихнуть в довольно глубокую потенциальную яму. Это можно сделать только двумя путями. Либо запихивать ядро в ОЧЕНЬ сильные гравитационные поля (как у нейтронных звезд), либо его окружить большим количеством адронов, т.е очень сильно сжать вещество, при этом сильно охладив.
Экзотический способ: окружить "тяжелыми" электронами, но они тоже распадаются. лол.
63 461745
>>61742

>хоть как-то повлияет на элемент "N"?


хоть как-то повлияет на элемент "N", причем именно в нужную сторону сдвинет вероятность?
64 461746
>>61744

>Чтобы замедлить распад


- он укорить хочет, как я понял.
65 461747
>>61746
Тогда ему надо наоборот вытаскивать из потенциальной ямы. Максимально разряженное вещество в отсутствие внешний полей.
66 461749
Всем привет. Имею тупой вопрос.

Предположим зимой чтобы согреть страну все 50 миллионов домохозяйств откроют настежь окна, заведут все свои автомобили да включат печку посильнее.
Потеплеет?
А если это сделает весь мир? Тогда потеплеет?
Если не сложно то с примерными рассчетами.
67 461751
>>61622

>Зачем организму нужен сон для восстановления мышц, тканей и тд? Чё он так не может?


Видел один ролик, там мужик задвигал теорию что во сне мозг переключается в другой режим работы. В этом режиме он проводит тонкую настройку всего организма. И он это даже якобы доказал в экспериментах на кошках. Он тыкал кошке в желудок и нейрон в мозге на это реагировал, но только во время сна.
Очень убедительно на мой взгляд он все расписывал, но похоже это какая то фриндж теория, т.к. нигде больше я ничего подобного никогда не слышал.
68 461756
>>61749
Гугли энергетический поток от солнца на удалении земли на метр квадратный, умножай на площадь какую посчитаешь нужной, страны, или половину поверхности земли, и сравнивай с количеством энергии, вырабатываемой в стране/на земле(она в любом случае вся уходит в тепло).
69 461758
Наткнулся на новость, что якобы удалось отредактировать геном взрослого человека, больного какой-то херней. Не очень понимаю, как можно отредактировать геном взрослого человека состоящего из охулиарда клеток. Двач, объясни
70 461759
Суп, нужно написать эссе на тему из физики/химии на 4к слов школяру на последнем году обучения страшей школы. Что поосветуете, чтобы было увлекательно, но и чтобы совсем не космический матан. Думал по поводу полупроводников, но не могу сформировать тезис, который можно бы было проверить эксперементально и чтобы он вообще хоть как-то интересным был. микросхемы чтоли паяльником греть и смотреть что будет?
71 461760
>>61758
А че такого, у тебя тельце постоянно обновляется, если в шаблон внести изменения, будет обновляться как тебе надо.
72 461761
>>61534 (OP)
Что такое спин?!
73 461765
>>61761
Число.
74 461767
скажите, что такое "эвристическое пренебрежение". Я туповат для такого. Желательно словами попроще.
75 461768
>>61726
Ты что дебил? Ты путаешь с турбовинтовым это раз, а два рекорд именно у обычного реактивного.
76 461769
>>61711
КПД двигателя внутреннего сгорания 40%.
Реактивный 20%.
/дискач.
78 461774
>>61760
не очень понимаю где находится шаблон? Я думал, он в каждой клетке и надо заменить его во всех клетках
79 461778
Меня занимает такой вопрос - вот есть всякие там вещества типа ЛСД или там ядов которые действуют в микроскопических количествах. Даже если взять банальную таблетку аспирина или сигарету. Как так получается что действующие вещества которых могут быть доли граммов попадают ровно туда куда нужно в туше под центнер, а не растворяются во всей ее массе. Разве это не удивительно.
80 461785
>>61774
А в чем проблема? Вирусы по твоему как работают
81 461803
>>61785
Тоесть вирус вот так просто может пропатчить ДНК одновременно и безошибочно ВСЕХ клеток организма(или хотя бы органа)?
Неужели это настолько просто и там нет хоть какой-то системы защиты?
Ели б было все так, то природный такой вирус смог был за пару часов превращать любое животное во что-то более схожее на инкубатор для вирусов.
82 461805
Так при передаче энергии по сверхпроводнику электроны вообще не текут?
И передача осуществляется чисто за счёт поля?
83 461806
>>61805
Ну т.е. она всегда осуществляется чисто за счёт поля, а течение электронов это чисто паразитный вредный процесс?
84 461807
>>61767

>4"эвристическое пренебрежение"


читаю статью Бэккера "Заметки к когнитивной биологии теоретической физики", там есть это словосочетание. Скажите пожалуйста, что оно значит, аноны!
85 461808
>>61806
Да. Электрический ток в любом проводнике осуществляется через поле. Но из-за то, что электроны или другие носители заряда взаимодействуют одновременно еще с другими полями, возникает "утечка энергии".
В сверхпроводнике, переносчики заряда уже взаимодействуют только с ЭМ полем или энергия взаимодействия переносчиков заряда находятся в разных неперекрывающихся диапазонах.
86 461809
>>61808

>В сверхпроводнике, переносчики заряда уже взаимодействуют только с ЭМ полем


Т.е. в сверхпроводнике другие переносчитки ЭМ поля, или они распространяются там, где нет других полей кроме ЭМ?
87 461812
>>61767

>эвристическое пренебрежение


https://en.wikipedia.org/wiki/Heuristic

>approach to problem solving or self-discovery that employs a practical method, not guaranteed to be optimal, perfect, logical, or rational, but instead sufficient for reaching an immediate goal


>mental shortcuts that ease the cognitive load of making a decision



heuristic neglect (ака medial neglect) - когда ты игнорируешь хуеву тучу механизмов, задающих твою когницию, и при этом не подозреваешь, что ты их игнорируешь. То есть, если тебе, например, эволюционно нахуй не сплющилось иметь доступ к своей нейрофизиологии, то для объяснения своих мыслей ты будешь подменять истинную каузальность какой-нибудь маня-метафизикой, и думать что всё норм.
88 461814
>>61812
Осторожно, этот пост (за исключением цитатки с вики) может исказить смысл слова. В частности, эвиристиками называют (намеренно неточные) "методы" принятия решений самим мозгом - никто никого не обманывает и не "подменяет" <...> "маня-метафизикой". Для казуалов советую почитать Канемана, а остальные сами найдут статьи на сколаре.
Ну и самое частое использование этого слова вообще с мозгом не связано и применяется в методологии науки (например, генетическое программирование в оптимизации).
89 461817
>>61814

>никто никого не обманывает и не "подменяет"


Речь об этом
https://rsbakker.wordpress.com/tag/heuristic-neglect-theory/

>This means that human metacognition and sociocognition are radically heuristic, systems adapted to solving systems they otherwise neglect.


>Human cognition possesses two basic modes, one that is source-insensitive, or heuristic, relying on cues to predict behaviour, and one that is source-sensitive, or mechanical, relying on causal contexts to predict behaviour.


>Not only are we blind to the enabling dimension of experience and cognition, we are blind to this blindness. We suffer medial neglect.



И об этом
https://www.academia.edu/1502945/The_Last_Magic_Show_A_Blind_Brain_Theory_of_the_Appearance_of_Consciousness
90 461835
>>61534 (OP)
Мужики, выручайте!

Проводились ли эксперименты по расчёту КПД искрового электрического разряда.

Ну то есть, вот есть сфера заряженная во столько то Кл, с таким то напряжением, её поднесли к другом шару, произошёл пробой, замерили количество Кл, шо перешло.
91 461846
В треде есть вторые Антоновы и Мигояны?

Как считаются реактивные двигатели? Вот хочу спроектировать ТРД с 1000 Н тяги. Как???
92 461848
Как определить глупый ли я есть ли тесты кроме iq?
93 461859
>>61805

>Так при передаче энергии по сверхпроводнику электроны вообще не текут?


Вполне себе текут. При этом они весьма хитрожопым способом связываются в пары.
Можно привести такую довольно натянутую аналогию. Представим себе длинный ряд столбиков. По нему не удобно передвигаться и можно запросто наебнуться. Но если очень удачно подобрать темп то можно легко бежать по ним как по земле и в хуй не дуть.
94 461861
Посмотрел тут видео про нагреватели
https://www.youtube.com/watch?v=J3pE_mDwCpk
И говорят, что у них у всех КПД 100%
Все 100% электричества, которые потребляет нагреватель выделяются в тепло и не важно есть там вентилятор или нет.
Это получается, что любой прибор в комнате, который включен в розетку выделяет все 100% энергии в тепло?

Если в комнате стоит телевизор, компьютер, 2 монитора, ноутбук и лампочка и они все в сумме потребляют 300 ватт, например, то они все в сумме выделяют 300 ватт тепла? И не важно как они устроены и что это за приборы, они выделяют всю потребляемую энергию в тепло? Или нет?
95 461863
>>61569
У тебя погрешность слишком высокая, возьми dt=0.01
96 461864
>>61861
Да. Те исчезающе малые проценты, что монитор переводит в оптическое излучение, в расчет не берем.
97 461865
>>61848
Ты на этой борде, в этом треде.
У меня для тебя плохие новости..
98 461867
>>61861

>Это получается, что любой прибор в комнате, который включен в розетку выделяет все 100% энергии в тепло?


В конечном счете да, но перед этим часть энергии он выделяет в виде чего-то другого.

Кстати, выходит КПД любого обогревателя 100%, ведь его полезная работа - и есть тепло. А ведь во всех учебниках твердят что 100% КПД нигде нет.
99 461869
>>61848

>Как определить глупый ли я


По обучаемости и усвоению инфы, по теме которая тебе реально интересна и которой хочется заниматься, в идельных окружающих условиях-в отсутствии внешних раздражителей и стресса.
100 461870
>>61867
Даже если брать обогреватель. всегда есть потери тепла, комната не абсолютно изолирована от внешнего мира, да в добавок тепловое излучение проходит через стены. Поэтому и говорят, что нет 100%.
101 461875
>>61867

>часть энергии он выделяет в виде чего-то другого


В виде чего?
102 461896
>>61875
По нагревательному элементу идет переменный ток, а значит он излучает, и это не тепло.
103 461904
>>61875
Звук/излучение/электрохимические реакции/термохимические реакции. В зависимости от градиента тепла возможно совершение механической работы, т.е банально будет раскручиваться вихрь воздуха/жидкости в среде. Дохуя возможностей проебать энергию.
104 461913
>>61869
А если меня депрессия/СДВГ(мне больше 20)/еще какие-то причины по которым я не усваиваю материал?
Если полностью их отбросить то выходит что выучится даже на что-то сложное может даже самый тупой, просто медленнее. А вот что-то новое изобрести - уже под вопросом.
105 461914
>>61870

>комната не абсолютно изолирована от внешнего мира


но ведь мы говорим об обогревателе, а не общей системе сохранения тепла.
Обогреватель как раз все 100% своей работы превращает в тепло. Или нет?

>>61875
Это если речь о не обогревателях а о других бытовых приборах.
106 461935
>>61904

>Звук


Значит 1 киловаттный динамик не выделит 1 киловатт тепла в идеально изолированную комнату?
107 461958
Аноны, дайте научным дисциплинам классы из ДнД, типо кто волшебники, тайные маги, водяные, огненные и проч. Понимаю, вопрос анрилейтед, но...
108 461960
>>61534 (OP)
Прочитал сегодня, в колбе компактной люминисцентной лампы содержится ртуть. Снимал как-то перегоревшую лампу, стекло треснуло, положил ее на полку, чтобы потом отвезти на утилизацию. Сидел в маленькой малопроветриваемой комнате с этой лампой около месяца. Тупой вопрос: я мог надышаться ртутными парами?
109 461961
>>61960
Очевидно что да, вопрос как много, и как это повлияло или может повлияет на здоровье.

Погугли действие ртути на организм.
Если оно проявляется сразу, и у тебя ничего за месяц не было-то понятно что ты нихуя толком не надышал и можно забить.
А если она не выводится из организма, имеет свойство накапливаться, и проявлять своё нахождение там черз несколько месяцев/лет - то думать дальше.
110 461965
Анон, я тян. Хочу сделать генеалогический днк тест. Отца и родственников по мужской линии нет вообще, поэтому полноценно это сделать не получится. Правильно ли я понимаю, что если у меня в будущем будет сын, то предков по мужской линии можно будет отследить только со стороны его отца, а о своих я нихуя никогда не узнаю?
111 461969
Жидкости и газы в однородном гравитационном поле должны разделяться на тёплую жидкость в верхних слоях и холодную в нижних. Даже в случае с одинаковой температурой по всему объему жидкости изначально. Ведь холодная жидкость тяжелее тёплой. То есть должна быть возможность "сепарировать" горячее и холодное.

Я прав?
Если да, то нарушает ли это второе начало термодинамики?
Что нужно знать, чтобы посчитать величину этого эффекта?
Наблюдается ли этот эффект где-либо?
Могу ли я назвать этот эффект в свою честь?
Можно ли этим объяснить температуру солнечной короны?
Если же я не прав, то в чём ошибка?
112 461971
>>61969

>Если же я не прав, то в чём ошибка?


Для этого ты просто берешь уравнения термодинамики и пользуешь их.
113 461972
>>61969

>Если же я не прав, то в чём ошибка?


В том что ты даже не разобрался что такое температура.
114 461973
Терпеть не могу физику этого мира.
Пытаешься забросить предмет на полку, а он отскакивает и падает.
Берешь хуйню из холодильника, она зацепляется за другую и все это говно падает на пол, да еще и куда-нибудь в говно укатывается.
Достаешь наушники из кармана, а провода все спутанные, да еще и с другим дерьмом в придачу.
Достаешь коробку, ее дно не выдерживает и все содержимое вываливается, тысяча мелких предметов, две тысячи из которых закатываются под кровать!

Почему нельзя было сделать нормально?
115 461974
>>61973
cosmological natural selection
Законы физики тонким образом настроены так, чтобы новые вселенные рождались из чёрных дыр
116 461975
>>61971

>Для этого ты просто берешь уравнения термодинамики и пользуешь их.


Какие из? Можно конкретнее?

>>61972

>В том что ты даже не разобрался что такое температура.


И что же такое температура?
117 461977
>>61975

>Какие из? Можно конкретнее?


Первый и второй закон.
118 461979
>>61975

>Высерает теорию


>Даже не умея пользоваться поисковиком в интернете


>И что же такое температура?


>Саентач, я открыл новый закон, хочу чтобы его назовали моим именем, как это сделать?

119 461982
Срочно напомните мне официальный биологический термин, который используется для обозначений несоответствий силы и массы насекомых при их потенциальном увеличении в размерах. Пиздец на языке крутится и не могу вспомнить слово.
120 461983
>>61982
Это не биологический термин, это закон квадрата-куба.
121 461985
>>61983
Не, там есть официальный термин.
Что-то там изм насекомых.
122 461986
>>61985
Хуйня какая-то.
Зачем нужен какой-то манятермин, если есть
общеприменимый закон.

Это прям как психология и психиатрия, пытаясь подмазаться к настоящим наукам, выдумывают манятермины и несуществующие заболевания, которые есть просто реакция на внешнюю среду и физические дефекты в мозге и нервной системе.
123 462019
>>61803
Вот именно что есть система защиты. Интерфероны, рнк-интерференция, приобретенный иммунитет и все такое. Не будь системы защиты, ничего бы не помешало вирусу пропатчить. Что они и делают и так, полагая сколько мусора у нас в днк. Если создать что-то на подобии вируса с нужной днк которая не будет тупо себя дублицировать, а будет чинить походить на своего и не триггерить охрану, то пропатчит весь организм. Плюс щас до того доходят, что могут патчить лишь нужные группы клеток - пример борьбы с раком. Только это все на стадии исследований.
124 462080
>>61986
То есть информатика не настоящая наука? Там тоже манятермины для несуществующей хуйни, которая просто является реакцией системы на введённые команды и физическим свойством архитектуры.
Так, стоп. А хули...
То есть физика высоких энергий не настоящая наука? Там тоже манятермины для несуществующей хуйни, которая просто является реакцией частицы на бомбардировку другими частицами. Всё можно редуцировать до слов "хуяк" и "пиздык"!
125 462083
>>61864

>Те исчезающе малые проценты, что монитор переводит в оптическое излучение, в расчет не берем.


Причем оно тоже восновном превращается в тепло, в конечном итоге
Те исчезающе малые проценты
фотонов - которые улетают в окно или превращаются в органические соединения при фотосинтезе в кактусе

>в расчет не берем

126 462107
Можно ли получать энергию из давления?
127 462108
>>61534 (OP)
Почему решая уравнение Пуассона в физике полупроводников полагают распределение плотности заряда ионизированной примеси непрерывным если один атом примеси на ходитсяв кубе с ребром в ~10нм?
128 462111
>>62108

>если один атом примеси на ходитсяв кубе с ребром в ~10нм


Потому что они очень маленькие, эти кубики.
129 462112
>>62111
10 нм это не очень то мало
130 462113
>>62112
Ну а какого размера берут полупроводник?
131 462114
>>62113
десятки сотни нанометров
132 462116
>>62114

>десятки сотни нанометров


Я думаю вполне себе макроразмеры берут. Поэтому и пользуются равномерным распределением.
133 462117
>>62116
Нет, активная область это десятки-сотнинанометров макимум
134 462118
>>62117
Объем какой?
135 462126
>>62118
100нм-250нм-25мкм например
136 462128
Почему у японок синяки на жопе, будто по 20ч в день за пекой сидят?
14270044442081071245.gif7,6 Мб, 370x207
Молния-Маккуин 137 462135
Почему электрический ток не может ударить в незаземленный объект, в то время как электрошокер бьёт человека вне зависимости от того, на чём он стоит, даже если на деревянном полу? Буду рад подробному сухому научному объяснению.
138 462136
>>62135
Потому что у шокера оба полюса рядом, а у молнии полюса Земля и трезубец Зевса.
139 462137
>>62135

>Буду рад подробному сухому научному объяснению.


I=U/R
140 462146
>>62136
>>62137
Хорошо, шокер не самый удачный пример. Как насчет пьезо-элемента от зажигалки? Почему она бьёт в палец, даже когда я стою на деревянном полу? Там только один проводок, с него сразу идет искра в тело.
141 462147
>>62137
Сухое, но совершенно не подробное.
142 462154
>>62135

>Почему электрический ток не может ударить в незаземленный объект.


Может.
>>62146

>даже когда я стою на деревянном полу?


Зажигалка ничего не знает о том где ты находишься.
Она просто из альтруизма делится с тобой своим зарядом.
Посколько дело происхит в воздушной среде а не в вакууме, то проходит дуга между пальцем и выводом с кристалла.
В вакууме был бы вакуумный пробой.
Даже если бы ты висел не касаясь земли.
Электричество само путь найдет, а лишнее - отгорит.
image.png40 Кб, 300x175
143 462162
>>62146

>Там только один проводок


No. Там ещё и масса. Само латунное сопло зажигалки либо корпус.
144 462165
>>62146
Этот
>>62154
все правильно сказал. (Хотя наверное стоит уточнить, что дуга/ток будет до тех пор пока потенциалы не уровняются, а это произойдет в зависимости от емкости - в случаи висящего в космосе Анона - это только его емкость, а в случаи заземленного его + емкость Земли.)

И да, еще уточнение

>пьезо-элемента от зажигалки ... Там только один проводок


- там всегда два "проводка" - обычно заостренный электрод внутри металлической трубочки.
145 462168
>>62107
В ДВС получают.
146 462170
>>62107
Просто из давления - нет. При перемещении чего либо под действием давления - да.
Работа = [расстояние] х [сила] = [давление] х [изменение объема]
147 462265
Что будет, если к электропроводке поднести магнетрон от микроволновки? Сгорит подключенная техника?
148 462270
>>62265
Наведенные поля могут вызывать изменение сопротивления проводки локальное. Может произойти кз проводки теоретически.
Если в технике нет предохранителя и она включена - может сгореть.
Если поднести магнетрон из микроволновки к включеной низкоточке, то выгорит скорее всего половина платы.
В печке все-таки Ватт 800. В упор слишком большие наведенные токи возникнут для тех сечений что на плате у проводящих элементов и микроэлектроники.
Это все мысленный эксперимент, сам я не проверял, лучше спросить в /ra.
149 462276
>>62270
А алюминиевый провод можно расплавить магнетроном?
150 462279
>>62276
Магнетрон это прежде всего 3-4 килограмма легкоусвояемого мяса 800 Ватт выходной мощности в виде излучения.

http://innowire.ru/tech/pochemu-ne-stoit-klast-metall-v-mikrovolnovku
151 462285
Нахуя мозг иногда запоминает сны? Это же информационный мусор. Ему не впадлу тратить ресурсы на интерпретацию и обработку этого буллшита?
152 462292
Бывает ли рак у растений?
Там ведь тоже клетки, которые делением размножаются. Бывает ли у них рак, когда клетки делятся без остановки?
153 462296
>>62285
Если у тебя мусор, это не значит что у всех.
154 462298
>>62296
А у тебя что, вещие сны?
155 462305
>>62298
В голове есть только то, что ты туда положил. И всевозможные комбинации, продуктивные или нет.
Если ты целыми днями смотришь на говно, то и снятся тебе реки говна, водопады говна, весь мир из говна. Может быть, надо что-то менять в своей ирл жизни?
156 462307
>>62305
Ты дурак и нихуя не понимаешь, о чем речь.
157 462308
>>62307
Пусть тебе сегодня приснятся дураки, анон. Чмоке.
158 462310
>>61769
КПД ионного - 80%
159 462311
>>61729
Погугли импульсные ядерные ракетные двигатели, принцип работы такой же.
160 462312
>>61761
Направление вращения электрона или любой частицы, хуй знает
161 462314
>>61761
Не задумывайся об этом. Это непредставимая категория.
162 462316
>>61761
Параметр квантовой природы, определяющий пространсвтенные ориентации и трансормации при взаимодейсвтии с полями, т.е группы преобразований. Путаница в названиях пошла из классической электродинамики.
163 462321
У теории чисел есть какие-то полезные применения? Кроме криптографии, конечно.
164 462338
>>62312
А с какой скоростью он вращается?
165 462339
>>62338

>с какой скоростью он вращается?


Это не совсем вращение. Потому что если попытаться прикинуть скорость, выйдет больше скорости света и какая-то чепуха.
15477022132160.png213 Кб, 420x420
166 462340
Как чёрные дыры могут быть вращающимися, там же время остановлено
167 462341
>>62321
Лол, это самая прикладная часть математики. Она дает целой ветви - численные методы.
168 462342
>>62340
С чего это время остановлено?
Ну, да время замедляется у объектов, которые находятся в глубокой потенциальной яме. Если сдвиг будет сильный, объект выпадет из наблюдения, это не тоже самое что время остановлено. Для объекта время будет идти своим чередом и даже есть время подумать перед тем как его разорвут приливными силами или ебнется в сингулярность.
Ах да, в физике под вращением понимается не само вращение как механическое движение, а физическая величина момент-импульса. В ТО является двухвалентным тензором, в квантовой теории становится особым объектом, называемой спинором.
134314351245.jpg34 Кб, 876x719
169 462344
>>62340
Время не останавливается, ты бы лучше спросил, как то у чего нет границы может вообще вращаться. понятно что световой конус будет не такой, это чет типо комфорного преобразования для спейс-тайм лайк ниар блэк хол
170 462345
>>62344

>С чего это время остановлено?


>>62344

>Время не останавливается


Всмыысле... Это ж по определению так
171 462347
Так... А откуда один объект/частица знает с какой скоростью относительно него движется другой/я объект частица, чтобы знать насколько замедлить своё время относительно его

И есть ли вообще реальное замедление врен, или только наблюдаемое?
35634563423.jpg36 Кб, 783x609
172 462349
>>62345

>Это ж по определению так


По определению все относительно.
Относительно тебя оно будет сильно замедленно, а оносительно черной дыры оно не остановлено.
на пиклриле просто гиперболические преобразовния, с черной дырой не связано. Для черной дыры выше.
173 462351
>>62349
Но она-то не перестаёт вращаться именно относительно нас
174 462352
>>62351
А у нас есть задокументированное доказательство? Типо видео черной дыры, которая вращается.
175 462353
>>62347
В Теории относительности время выделяется в дополнительное измерение. Так получаем единое четырех мерное пространство событий, оно такое же как и наше привычное, только вот точками являются событиями, а расстояния между ними интервалы. Главная особенность интервала в том, что он может из-за особенности времени иметь нулевую величину или даже отрицательное величину. Т.е расстояние между двумя неравными точками нулевое. В пространстве событий совокупность всех точек с нулевым расстояниями называют изотропной поверхностью ну или частный случай световой конус.
Физический смысл светового конуса в том, что он является единственной действующей реальностью, "НАСТОЯЩИМ", "СЕЙЧАС" для взятой нашей точки.
В пространстве событий возможны преобразования(гиперболическое вращения) при котором интервалы сохраняются, т.е можно повернуть оси так, что световой конус не измениться и следовательно физика не измениться. Собственно и это называются принципом относительности.
Движение нашей частицы, будет представлять некую линию в пространстве событий. Касательная к этой линии является вектором скорости. При помощи магии дифгеометрии можно выделить параметр от которого вектор скорости будет единичным по длине и задавать направления. Этот параметр называется собственном временем. Вот так получаем, что вектор скорости в трехмерном задает направление в пространстве событий, т.е наши оси. И следовательно преобразование скоростей есть наш поворот. Тут важно еще момент, что собственное время определенно только конкретной точки, при преобразований скорости параметр будет меняться. Это изменение и определяет замедление времени.
На счет сокращение длин, тут все просто: сокращаются проекции на трехмерное пространство.

И да, поскольку для данного момента времени существует только одна реальность, замедление времени наблюдаемая реальность.
176 462354
>>62352

>задокументированное доказательство


>черная дыра


Кхе, кхе.
177 462355
>>62352
Понимаешь, чёрную дыру невозможно увидеть, в принципе.
Можно увидеть как она что-то засасывает, но чтоб было хорошо видно, нужен видос засасывания звёзд и нет, таких нет пока.
178 462356
>>62354
Я тебе про это и говорю. С чего ты взял, что

>Но она-то не перестаёт вращаться именно относительно нас


>Время останавливается по определению

179 462357
>>62356

>Но она-то не перестаёт вращаться именно относительно нас


Это потому что впринципе говорят о вращающихся чёрных дырах, эффект от вращения учитывают при всевозможных расчётах и он вроде как подтверждён.

>Время останавливается по определению


А это потому что это впринципе свойство чд, она не выпускает свет, а дальше подтягиваются все взаимосвязи свойств пространства-времени, замедления времени в зависимости от скорости движения относительно скорости причинности в пространстве-времени, замедления времени из-за искривления пространства, замедлении времени из-за гравитации(ну тут кароч много тавтологий понаписывал, но пусть будут).

Свет не выпускается-следовательно искривление пространства/гравитация такая, что время останавливается, т.к. заредление считается через разность от скорости света.
Чёрная дыра=по определению то в чём время остановлено.

Ну и плюс все везде об этом говорят.
180 462358
>>62355
А это тут причём?
И вроде как есть.
181 462359
>>62357

>она не выпускает свет


Вот тебе пища для размышления: если пустить фотон по нормали, т.е перпендикулярно "поверхности" (горизонту событий) от черной дыры, то куда и с какой скоростью он улетит. Даже можно взять массивную нейтронную звезду в верхнем пределе масс, и для него пустить фотон перпендикулярно его поверхности.

>Это потому что впринципе говорят о вращающихся чёрных дырах


Говорят так потому что момент импульса, которым обладала звезда-предшественник черной дыры, должен сохраняться. Я и тебе так же дал пищу для ума: как объект без поверхности (математический, так как мы не знаем его структуры) может вращаться и иметь момент импульса.
Вообще время не течет медленнее или быстрее в каких то областях, оно течет медленнее или быстрее относительно других СО. И есть увлекательный факт, скорость света в любой системе отсчета равна скорости света (попробуй его применять к первой задачке).
У тебя в твоем описании много неправильного понимания физики относительности.
182 462360
>>62357

>Свет не выпускается-следовательно искривление пространства/гравитация такая, что время останавливается, т.к. заредление считается через разность от скорости света.


>Чёрная дыра=по определению то в чём время остановлено.


На словах не принимается. Решай уравнения ОТО смотри. Хотя о чём я, уже всё давно решили и в учебники записали.
183 462361
>>62360
Т.е. на поверхности объекта, на поверхности которого первая космическая скорость будет равна 300т. км/с время не будет замедленно относительно наблюдателя, находящегося на объекте, первая космическая на поверхности котоогон будет условно 0, и неподвижного относительно первого объетка, замедление времени не будет стремиться к бесконечности?
Это что-то новенькое.
184 462362
>>62361

>замедление времени не будет стремиться к бесконечности?


Там хитрости всякие возникают.
185 462379
Сап, наукач. Молодой талантливый автор фентези на связи, есть у меня один мирок устроенный довольно просто - есть три объекта: 1) населенная "планета" где происходит основное действие 2) местное дневное светило 3) местное ночное светило.

К сожалению я ни хрена не шарю в ваших математиках и физиках - в целом это не помеха, ибо мирок сотворен как и положено местными богами и на реалистичность законов природы не претендует от слова совсем.

Но тем не менее один вопрос никак не могу обойти, нужна подсказка. Загвоздка в том, что эти три объекта по замыслу равны друг другу - и по размеру, и по взаимному расположению. То есть это не система звезда - планета - спутник, они все три должны как-то крутится друг вокруг друга (общего условного "центра масс", для простоты можно сказать что массы равны). Вопрос же о том - по каким траекториям,чтобы имело место хотя бы подобие земных смены дня и ночи? Я так понимаю местное солнце будет вставать постоянно в разных местах, и от оси вращения "планеты" ничего не зависит? Смены сезонов и разных климатических поясов тоже не будет (это то что мне и надо)? Насколько продолжительными будут периоды когда на небе планеты видны оба светила, возможны ли в таких условиях затмения? Кто нибудь может захуярить модель (многого хочу, да, но вдруг)?
Three-bodyProblemAnimationwithCOM.gif501 Кб, 300x300
186 462384
>>62379
Ты замахнулся на одну из не решенных математических проблем - Задача трех тел. В аналитическом и общем виде задача не решаема, и даже в приближении нет стабильного решения.
Хотя всегда можешь обмазываться численным методами.

Да, в целом твоя система будет очень не устойчивая, приливные силы у тебя на месте же? Могут быть даже такое, что светило остановится, а потом обратно пойдет. Естественно ни о какой смене дня и ночи, тем более сезоном не может идти речи.
187 462385
>>62384

>Естественно ни о какой смене дня и ночи, тем более сезоном не может идти речи.


Может симметрия какая существует, когда решение стационарно?
15498818647880.jpg36 Кб, 500x427
188 462386
>>62379

>талантливый

189 462387
>>62385
Нет, стабильные решения существуют только для частных случаев и то массы объектов не должны быть ровны.
190 462388
Подскажите где можно поискать книги по биологии в открытом доступе интересует систематика , нужны достаточно новые работы.Да и вообще есть какие нибудь аналоги рутрэкера для научной литературы?
191 462394
>>62379
И нахуя тебе это, если у тебя нереалистично все. Пусть тогда эти 2 солнца вращаются вокруг твоей планеты с противоположных сторон, а планета в центре, а всю эту систему пусть стабилизирует сущность в виде гномика из 4 измерения, или какая нибудь темная материя.
192 462419
>>61534 (OP)
Я знаю, что интерпретация квантовых явлений это платина, но может объяснить мимодовену, почему невозможность определения всех элементов суперпозиции объясняется свойствами МАТЕРИИ (типа на квантовом и микроуровне они разные, или даже многомировое объяснение), а НЕ свойствами ПРОЦЕССА ИЗМЕРЕНИЯ (т.е. реальна суперпозиция, а наблюдать можно только её так сказать грань, типа как мы изменяем параметры тока мультиметром по очереди, а не одновременно, но это же не значит, что в момент измерения силы напряжение отсутствует, плюс, понятно, прибор тоже взаимодействие оказывает)?
(Интересует скорее не сам ответ, а материалы, которые могут к нему приблизить).
КФ, по которой даже близко консенсуса нет, для чайников: платиновый реквест; но всё же.
193 462420
>>62419

>определения всех элементов суперпозиции


Что это значит?
194 462421
Вот смотрите. Микромир случаен целиком и полностью. Так какого хуя макромир не случаен?
195 462422
>>62420
Это то, что не получается сделать из-за коллапса волновой функции (как я понял).
196 462423
>>62421
он не случаен, не в твоем понимании по крайней мере
197 462426
>>62419

>почему невозможность определения всех элементов суперпозиции объясняется свойствами МАТЕРИИ


Я думаю тут всё дело просто во взгляде. Какая разница, свойство материи это, или свойство процесса измерения. Это как из постоянства скорости света следует геометрия пространства, или наоборот из геометрии пространства следует постоянство скорости света. Скорее всего опираются на фундаментальные волновые свойства, которые уже и дают все эти ограничения.
198 462427
>>62421

>Так какого хуя макромир не случаен?


Макромир - усредненный микромир.

>Это то, что не получается сделать из-за коллапса волновой функции


Хуета какая-то
199 462429
>>62423
Ты же понял мой вопрос. Хули выебываешься?
200 462431
201 462434
>>62427
Но почему мы можем предсказывать это усреднение?
202 462435
>>62434
Центральная предельная теорема
203 462443
>>62384

>Задача трех тел.


Спасибо. Восьмерка Мура же вроде как раз подходит для случая с равными массами?

Сезоны мне как раз и не нужны, я уже писал. Вот то что смена дня и ночи по пизде идет, это хуже, придется вводить не привязанный к движению миров способ измерения времени.

>>62394
Определенная степень достоверности должна быть. Если свою маняфизику я могу закрыть любым допущением и стабилизировать систему хоть гравитационными волнами, хоть темной сущностью, хоть божественной волей, хоть натяжением мирового эфира, то местная астрономия должна хотя бы выглядеть законченной и позволять представить картину мироздания из глаз местных обитателей, а читатель/зритель/игрок должен иметь возможность сравнивать с его представлениями о реальности. Не имея определенности для себя как это выглядит - я просто наворочу взаимоисключающей хренотени (те же день и ночь запросто оставил бы). И пизда погружению в мир.
204 462445
>>62341
Численные методы, это же про матан, дифуры и прочее говно. Каким боком тут теория чисел? Можешь на примере каком-нибудь пояснить связь?
205 462456
>>62388
Видимо это не слишком тупой вопрос и тем не менее.
206 462459
>>62339
Если это не вращение тогда что это?
pikachu-clipart-png-icon-720483-3218447.png31 Кб, 640x480
207 462460
Я понимаю, сейчас начнётся ИДИ НАХУЙ СО СВОИМИ ВЫДУМАННЫМИ МИРКАМИ, но всё же. Почему электрический тип покемонов наносит уменьшенный урон травяному типу покемонов? Какбы, если молния в дерево херакнет, то его разнесёт, если в поле, но на траве выжженный след останется.
208 462461
>>62460

>травяному типу покемонов?


По сравнению с кем?
См. электрическое сопротивление различных тканей животных и растений.
209 462464
>>62459
Свойство, которое показывает как меняется волновая функция при вращении системы координат.
210 462468
Так, успокойте меня, скажите что-нибудь простенькое, благодаря чему мне перестанет так выносить мозг из-за факта того что электромагнитное поле может иметь чуть ли не бесконечное количество колебаний разной(и не разной) длины в одном и том же объёме, при этом совершенно никак друг на друга не воздействуя...
211 462470
>>62468

>совершенно никак друг на друга не воздействуя


Совсем чуточку друг с другом фотоны ебутся.
212 462471
>>62470

>Совсем чуточку друг с другом фотоны ебутся.


Ну как-то не успокаивает...
213 462473
Какие вообще есть доказательства того что поля квантуются?
214 462476
>>62473
Существование квантов. Внезапно да?
215 462477
>>61534 (OP)
Почему м-теории считается говном? Там же все доказано уже
216 462478
>>62476
Ты просто переписал мой вопрос, это то же самое, это не ответ.
217 462480
>>62478
Самый простой эксперимент, уменьшение интенсивности лазера, рано или поздно начнешь замечать порционность света. Энергия от лазера с частотой v всегда будет кратна hv.
218 462481
>>62477

>Почему м-теории считается говном?


Никто не считает ее гавном, кроме /sci

>Там же все доказано уже


Не доказано.
219 462484
>>62461

>По сравнению с кем?


Например, по сравнению со льдом, камнем, тьмой хотя тьму то должно освещать. Но по сути то, в нормальный урон запихали те типы, которые в дошкольной программе по физике не пересекаются друг с другом, поэтому урон зависит уже от статов самого покемона.
Уменьшенный урон электрические помимо этого наносят электрическому, никакого урона земляному, повышенный урон воздуху и воде, сам не получает урона от стального типа.

>См. электрическое сопротивление различных тканей животных и растений.


Может, если электричество благотворно влияет на растения, то перед тем, как словить разряд в ебало, травяные попросту обмазываются ионизированным воздухом? Ведь трава побеждает воду и землю за счёт того, что пожирают их, и с электричеством так, но до тех пор, пока по жопе разряд не пройдётся. Сам на свой вопрос ответил похоже, сорян.
220 462485
>>62456
Ты с такими вопросами лучше к медикам обращайся
222 462498
>>62480
Берём поток воды и начинаем перекрывать его так часто, как можем технически, в результате чего вода начинает литься "кучками". Тем самым доказываем что вода состоит из элементарных водяных частиц(не кучек).

Можешь объяснить почему твой пример не равносилен этому?
223 462499
В импульсных трансфораматорах по сравнению с обычными просто микроскопических размера сам трансформатор. И типа это достигается за счёт большого увеличения частоты работы(по сравнению с 50герц в эл. сетях).

Но я не врубаюсь, и что что повышение частоты, всё равно в конечном итоге через маленький трансформатор в импульсном блоке нужно пропустить такую же мощность, как и через здоровенный классический трансформатор той же мощности. Как так?

Или при сильном увеличении частоты работы у трансформатора пиздецки возрастает КПД?
224 462505
>>62499
Все просто, с увеличение частоты увеличивается плотность магнитного потока, на низких частотах это достигается за счет магнитопровода, на который рассевается энергии больше, чем на омическое сопротивление обмотки(если мы рассматриваем силовые трансформаторы).
Да, чем выше частота, тем лучше трансформатор трансформирует ток. Да и размерами меньше выходит, поэтому сейчас их везде пихают, особенно с распространением мощных полевых транзисторов. В прочем тут есть и свои недостатки вроде ебучих НАВОДОК на сигнальные цепи.
Однако высокие частоты не используют на крупномасштабных линиях, ибо возрастают потери при передачи по длинных линий. Так что частоты 50-60 Гц это компромисс на крупных электросетей.
225 462507
>>62498
в одном случае ты перекрываешь трубу периодически, в другом нет.
226 462510
>>62507
Так где доказательство что в том примере "замечания порционности света" будет означать что видя одну из этих порций-видишь именно квант света, фундаментально малый, а не то что "размер порции"-просто технический предел установки делать минимально малые порции, только для её возможностей?
227 462511
>>62505
Спасибо, всё понял.
228 462514
>>62510
Никто и не говорит что ты видишь именно "квант", ты видишь что энергия поля квантуется, то есть меняется на дискретные значения. И твой вопрос бы про то что поле квантуется.
229 462515
>>62514
А, теперь понял что имелось ввиду, типа если бы вода не состояла из элементарных водяных частиц, то и кучки бы отдельные из неё не могли бы получиться, они бы переходили из толстой кучки, во всё утоньшающуюся, всё тоньше и тоньше, а затем опять на расширение до следующей, максимально толстой части следующей кучки, и опять на сужение..

Но вмё равно как-то неубедительно. Где доказательство что кажущаяся дискретность только и является кажущейся, из-за несовершенства приборов которыми на этот процесс смотрят?
230 462517
>>62515

>из-за несовершенства приборов которыми на этот процесс смотрят?


Пока нет ни одного известного физического процесса, где возникал бы более совершенный прибор.
231 462518
>>62515
Существование фотонов, если тебе про эти кванты, еще Боте в свом опыте показал.
232 462531
>>62517
И что за прибор?
image.png201 Кб, 750x300
233 462534
Можно ли теоретически просчитать третичную структуру белка?
Есть такая задача в математическом виде? Или какая-нибудь похожая задача, решение которой поможет решить эту задачу.
234 462535
>>62473
Поля не квантуются. В современной квантовой теории поля - стандартной модели, поле это некая среда, заполняющая все пространство и обладающее бесконечно большим числом степеней свободы на единицу объема. Возбуждение поля описывается тензорами, векторами.
Еще раз, в современной стандартной модели нету элементарных частиц (единых неделимых объектов, квантов), вместо них тензоры, возбуждающие поле. В современной стандартной модели нету волновой функции. Вместо волновой функции там полевой оператор.
Короче говоря, всё это сложно. Квантовая теория поля не хер собачий. Изучайте квантовую теории поля. Все ваши институтский совковые знания про "волновые функции" и "кванты" уже очень сильно устарели.
235 462537
может один человек чувствовать себя в одной части планеты лучше, чем в другой?
к примеру, бывало, что у горных жителей, легкие и сердце больше, чем у тех ,кто живет на низкой высоте, где плотность кислорода выше?
как влияет дождливая среда на пустынных жителей и на оборот? чувствуются перепады давления перед дождем или они слишком малы?
холодно большую часть года или тепло большую часть года?
236 462539
>>62535

>Квантовая теория поля


Т.е. просто удобная условная моделька для расчетов, которая доёт хорошие результаты?

Это официальная версия, а то что постят везде в инэте её название как истинное описание материи(квантовое), а не просто условность-модельку для расчётов-мракобесие недалёких и повеихностных журналистов-редакторов?
237 462540
>>62534

>Есть такая задача в математическом виде?


Есть конечно, но ни один комп не потянет такое.
239 462542
>>62539
Абсолютно все научные теории это модели. За истинной пиздуй в /re
240 462543
>>62535
Ну ты шизик, только с другой стороны вещаешь. Это как предел -0 и +0.

>Квантовой теории поля


которая вся строится математикой вторичного квантования.
241 462544
>>62543
Всё иди нахуй. Это тред ответов на тупые вопросы. Это не тред споров образованного человека (т.е. меня) с тупым дибилом (т.е. тобой). На изначальный вопрос я ответил. Поля не квантуются.
242 462545
>>62544

>Поля не квантуются.


Пруфов же конечно не будет. Ну ты идиот, поздравляю. То что ты там напиздел про операторы, ничего не говорит.

>Это тред ответов на тупые вопросы.


Ну значит мое замечание не такое тупое. Во-вторых, если это тред ответов на тупые вопросы, это не значит, что можно пиздеть все нипочем поподя.
Самое абсурдное, что может сказать человек в 21 веке - поля не квантуются.
243 462546
>>62544

> Операторы с собственными значениями


> Поля не квантуются.


=_= Это какой-то тонкий троллинг?
244 462549
>>62542
Вопрос был не в этом, а в том как этот принцип(квантовости) этой модели позиционируется, как просто для упрощения для расчётов, или с замашкой/утверждением что именно так оно и есть.
245 462551
>>62545

>Поля не квантуются.


>Пруфов же конечно не будет.


Т.е. никаких экспериментов прямо/косвенно подтверждающих/опровергающих это не проводилось?
Я это хотел узнать.
246 462553
>>62551
Кароче, ответ один - квантуются. КТП это просто формализм, который один шиза неправильно понял - там все квантуется. Эксперементов подтверждающих квантовость микромира тоже уйма.
Вообще утреврждение типо "экспериментов прямо/косвенно подтверждающих/опровергающих это не проводилось?" совсем неверное. Это предполагает, что была построена математическая квантовая теория, и только потом начали искать ее подтверждение в эксперементах. Все было наоборот, были необъяснимые с классической точки зрения эксперементы и только квантовая теория начала их номрально описывать и самое главное предсказывать. Предсказываемость - один из главных параметров правильной теории. например многие шизонаучпоперы с алтернативными теориями только описывать, т.е подгоняют свое описание под уже существующие данные, но ничего нового предсказать не могут.
247 462562
>>62553

>Кароче, ответ один - квантуются.


>ЯСКОЗАЛ


>Эксперементов подтверждающих квантовость микромира тоже уйма.


>Не привёл ни одного, зато куча текста пустой воды о его важном МНЕНИИ по этому вопросу


Ну не знаю...
248 462563
>>62549
Что значит "так оно и есть"? Еще раз повторяю, за истинами обращайся к религиозным учениям. Каждая признаная научная теории справедлива в пределах своей применимости. Механика Ньютона 17 века, это верная научная теория. Про неё можно сказать "так оно и есть", только при скоростях много меньше скорости света. Теория относительности Эйнштейна расширяет этот предел. Более точно описывает мир, при определенных условиях (при околосветовых скоростях). Это тоже справедливая теория, и про нее можно сказать, что "так оно и есть", но только если дело не касается микромира (10^-9 и менее). Современная квантовая теория поля (стандартная модель) с очень высокой точностью описывает этот микроуровень. Но и она имеет свои границы применимости. Ни одна из приведенных выше теорий не описывает мир с абсолютной точностью. Улавливаешь?
249 462564
>>62562
У людей реализм головного мозга. От этого страдали даже великие физики начала 20-го века. А ты пытаешься это объяснить мимокроку. Это сложно принять сразу.
250 462565
Ладно, пожалуй снизойду до вашего уровня и поясню вам элементарные вещи.
Определение поля в физике - это некая среда, заполняющая все пространство и обладающее бесконечно большим числом степеней свободы на единицу объема. Уже из этого определения тривиально доказуемо то, что оно не может квантоваться. Но я примерно представляю в чем тут ваша загвоздка. Вы вероятно думаете, что поле это некий объект, который существует в реальности. НЕТ, это не так. Понятие поля, это просто математическая абстракция, введенная учеными для более точного описания своих теорий.
Понятия о натуральных числах, множествах, окружностях прямых все это не реальные объекты. Все это математические абстракции. Без которых не мыслима современная математика и вся построенная на них физика. Кстати само по себе понятие "пространство, это тоже чисто математическая абстракция.
Господи, да с кем я блядь тут вообще общаюсь, как можно не понимать таких элементарных вещей?!
252 462567
>>62565

>бесконечно большим числом степеней свободы на единицу объема. Уже из этого определения тривиально доказуемо то, что оно не может квантоваться


Про гильбертово пространство ты не слышал видимо...
Это такой тонкий троллинг? Или ты слово "квантуется" как-то по-своему понимаешь? При квантовании сохраняется бесконечное число степеней свободы. У электромагнитного поля, например, бесконечное и несчетное количество частот. Но измеряя амплитуду поля при какой-то конкретной частоте, ты всегда будешь натыкаться на какое-то целое кратное этой частоты. Вот это число заполнения данной частоты и называют числом фотонов.

>как можно не понимать таких элементарных вещей?!


Ты уходишь в очевидности уровня "это всё модели этого нет в реальности", но когда человек спрашивает "существуют ли молекулы в реальности" он ждет не философии науки с "это всё модели бла-бла-бла", а несколько другого ответа.
253 462568
>>62564
это сюда
>>62563
254 462569
>>62567
Таки я не понял, поле это математическая абстракция или нет?
255 462570
>>62569
Всё математическая абстракция, так как и нейтроны и протоны тоже в поле.
256 462571
>>62563

>Улавливаешь?


Улавливаю что опять куча воды не к месту, говорю же, вопрос не об этом был, там конкретно описано, вопрос в том, заявляет ли квантовая теория, или те кто её создали, или те кто в ней серьёзно разбираются, что мир/поля квантовые, или это просто теория, в которой расчёты ведутся "квантово", что позволяет получать хорошие результаты.

Плюс читай вопрос про эксперименты.
257 462572
>>62570
Если всё вокруг это математическая абстракция, то какого хуя я тут осознаю, что общаюсь с хер знает кем на двачах? Какого хуя я сейчас ощущаю клавиши клавиатуры под своими пальцами? Какого хуя я чувствую кресло своей жопой. Что в этом мире не математическая абстракция? Что реально?!
258 462573
>>62571

> что мир/поля квантовые


> расчёты ведутся "квантово"


Это по сути одно и тоже. Никакого вопроса тут нет, а очередной байт на вопрос типа сколько ангелов умещается на кончике иглы.
259 462574
>>62572

>Что реально?!


Что такое реально?
260 462575
>>62574
Я не знаю, ты мне объсни. Нахуйсмы тут вообще обсуждаем какие то научные теории, если мы вообще недоговорились о начальных аксиомах? Какие они начальные аксиомы? Что есть реальность? От чего вообще отталкиваться?
261 462576
>>62575

>От чего вообще отталкиваться?


От теоремы Гёделя о неполноте!
262 462577
>>62575

>Что есть реальность?


И без этого наша цивилизация существует, строит, изобретает. Вопрос избыточный. Я бы даже сказал ненужный, сродне вопросу о "смысле жизни", т.е просто неудачная конструкция языка.
263 462578
>>62577
Ответ неверный, подумай еще.
264 462579
>>62578
Пошел нахуй, сходи еще раз.
265 462580
>>62573

>Это по сути одно и тоже.


Нет, не одно и то же, хватит философствовать, это не ph/ а sci/, всё-таки, каким бы он не был.
Когда методом конечных элементов считают что-то, то тем самым не пытаются доказать что общитываемый объект из этих конечных элементов и состоит. И метод конечных элементов это не утверждает и считающий это не утверждает.
Вот я и спрашиваю, утверждает ли квантовая теория поля что поля состоят из квантов, или она просто принимает такое допущение для расчётов.
И существуют ли какие-то эксперименты, которые могли бы это доказать или опровергнуть, проволились ли они, если нет то почему, и какие результаты были.
Читай вопросы, а не философствуй, за философией в ph.
266 462581
>>62580

>проволились


*проводились ли
267 462582
>>62576

>От теоремы Гёделя о неполноте!


О, кстати, какие есть философские, ну или не совсем философские теории, которые тем не менее определяют что-то такое фундаментальное в науке?
Что-то типа фундаментальных научных вопросов/ограничений.
268 462583
>>62562
Если человека в гугле забанили, то мне похуй. Я привел один эксперемент Боте, доугой анон про лазер сказанул. Еще давай про чернотельное излучение, two slit вообще уже попса. Если тебе только посраться на пустой почве охота, ну похуй гоу, только ты тут выйдешь долбаебом полностью покрытым дерьмом.
Я сказал важную вещь из которой вся эта хуйня с подтверждением квантовой теории просто смывается в толчек:

>Вообще утреврждение типо "экспериментов прямо/косвенно подтверждающих/опровергающих это не проводилось?" совсем неверное. Это предполагает, что была построена математическая квантовая теория, и только потом начали искать ее подтверждение в эксперементах. Все было наоборот, были необъяснимые с классической точки зрения эксперементы и только квантовая теория начала их номрально описывать и самое главное предсказывать.


Это на 3м году обучения на курсе общей квантовой физики мусолится. Следовательно, если тупой, то берешь учебник и читаешь.
269 462584
>>62563

>Механика Ньютона 17 века, это верная научная теория


Особенно для орбиты Меркурия она верна.
270 462585
>>62584

> справедлива в пределах своей применимости

271 462586
>>62585

>Про неё можно сказать "так оно и есть", только при скоростях много меньше скорости света

272 462587
>>62586
Мне кажется ты доебался просто чтобы доебаться.
273 462589
>>62586
edit: орбитальная скорость Меркурия 0.0001578с
>>62587
Твоя суть была верна в принципе. Только вот теория Ньютона не верна, и нихуя даже в пределе она не верна, она очень хорошо совпадает. Даже если смотреть относительно. Так же, если найдут лучшую теорию чем ОТО, то она будет нихуя неверна. В этом и есть весь смысл.
274 462590
>>62589
Просто доебал уже релятивизм ебучий на /сосай.
275 462591
>>62583
Пример с лазером я типа обосрал, и мне толком никто ничего не сказал почему не обосрал, а сам обосрался сделав это.
Другие эксперимент посмотрю и отвечу.
276 462592
>>62589

>она очень хорошо совпадает


Эм. Это можно сказать вообще обо всех теориях на свете, даже о тех, которые еще не были придуманы.
277 462593
>>62589
А, вы про то что ни одна из теорий не говорит как оно там на самом деле, а просто помогает точнее считать?

Да не
278 462595
>>62565
Ты пишешь что все это математическая абстракция и говоришь, что поля не квантуются, хотя даже в этой математической модели они квантуются. Что вообще ты хотел сказать? Я просто понять не могу. В данном формализме полевом все прекрасно квантуется. Этот формализм лучше всего описывает и предсказывает современный мир. Что еще надо человеку, чтобы не прослыть шизиком в современном мире, так мало.
Ты в сай пришел а не в фай.
279 462597
Поля не квантуются
280 462598
Толи все кваантуется, толи не квантуется. Вообще нихуяне понятно. Где в этом мире истина? Кругом какие то манятеории шищиков, а мне хочется абсолютного нерпреложного знания? Как оно на самом деле то?
Kj2MTbdvHz8.jpg406 Кб, 1280x785
281 462599
282 462600
>>62598

>Как оно на самом деле то?


А что, если я тебе скажу, что ты в жизни не видел ни одной вещи. Ты видел свет от вещей, но никогда не видел вещи.
283 462601
>>62600
Я просто хочу дергать письку и играть в доту. Ну хули все так сложно устроено то? Ну почему нельзя было попрощ?
284 462602
Когда будет полноценная аугментика, превосходящая по своим качествам биологические органы?
285 462603
>>62595

>хотя даже в этой математической модели они квантуются.


Пиздец ты в логике силён, уважение.
286 462604
>>62602
В 2027 году
287 462605
>>62604
С чего ты взял?
288 462606
>>62603

>Пиздец ты в логике силён, уважение.


А ты нет.
289 462607
>>62606
Не рвись.
290 462608
Вот, когда программа на компьютере по мат-моделе считает движение потока жидкости/газа, условно заменяя миллиарды частичек реальной среды тысячей-другой более крупных, ни что не мешает взять например в два раза больше в два раза более мелких частичек и посчитать точнее. Но смоделировать каждую молекулу в потоке мешает нехватка вычислительных мощностей.
А в квантовой теории что мешает взять меньший квант для расчётов? Как там определяется фундаментальный(для теории) квант?
291 462615
>>62608
Поток жидкости обсчитывается при помощи дифференциальных уравнений в частныз производных.
Квантовая теория поля - оперирует вариационными(функциональными) уравнениями, так что строить тут аналогии гиблое дело.
292 462621
Как получается, что один химический элемент может превратиться в другой прямо как во влажных мечтах алхимиков?
293 462622
Я думал, что тут есть отдельный тред по величайшему изобретении столетия - русскому антигравитационному двигателю Леонова, а вы тут хуйней маетесь.
294 462628
>>62621
там в ядерных реакторах ртуть в золото превращали. погугли в эту сторону
295 462643
с помощью NIR спектроскопа получаю график проходимости тех или иных частот. есть база NIR спектрограмм молекул. делаю спектрограмму куска еды и хочу узнать, сколько в нем жира и протеинов? спектрограммы молекул жира у меня есть. как я увижу по спектру еды в которой много разных молекул, что какой-то пик на спектрограмме, это жир?

для домашней лабы NIR дешевле всего? для определения состава еды, поиска токсинов, сахара в крови.
296 462644
>>62643
Эээ... За стремление конечно тебе лайк. Но по описанию кажется, будто ты пытаешься плоскогубцами забивать шурупы.
297 462649
>>62602
брумп, куколду с -5 неприятно

>брумпать в тематике

298 462654
>>62621
Бамп вопросу
Sage 299 462685
>>62654
Свойства атома зависят от заряда ядра и количества электронов на орбиталях. Меняем эти параметры - получаем новые свойства
Sage 300 462686
>>62649
Когда она станет легкодоступна и реально нужна. Сейчас нет профессий или занятий, для которых тебе обязательно потребуется, например, поставить себе гусеницы вместо ног. Когда возникнет спрос - тогда и появится предложение.
301 462687
Сажа ебаная прилипла, сори
33.JPG23 Кб, 496x291
302 462688

>Как выяснили ученые, температура поднимается на 3 градуса каждые 100 метров вглубь Земли. Эта цифра является постоянной для всех континентов и частей земного шара. Такой рост температуры происходит в верхней части земной коры, примерно первые 20 километров, далее температурный рост замедляется.



Это значит, что если я пробурю скважину в 400-500 метров, загоню туда U-шланг с утеплением и оголением на последних 20 метрах - я смогу отапливать дом на халяву?

почему так никто не делает?
303 462689
>>62688

>пробурю скважину в 400-500 метров


Поэтому и не делают
304 462692
>>62688
Потому что глубоко. Но там где подземные воды сами выходят на поверхность, их как раз используют в качестве отопления. Гугли геотермальная энергия.
305 462707
Как с первой попытки попасть в ваковский журнал? Кому забашлять?
nasha1.JPG40 Кб, 463x349
306 462710
Но как это вообще возможно, как может быть устроена Природа, чтобы квантово сцепленные частицы мгновенно взаимодействовали друг с другом в независимости от разделяющего их расстояния, будь они хоть в разных концах вселенной?
307 462712
>>62710
А это точно так?
308 462714
>>62712
А то.
However so-called "loophole-free" Bell tests have been performed in which the locations were separated such that communications at the speed of light would have taken longer—in one case 10,000 times longer—than the interval between the measurements
https://en.wikipedia.org/wiki/Quantum_entanglement
309 462715
>>62688
В смысле, никто не делает? У меня дом от теплового насоса обогревается, грунтовый теплообменник длиной 480м на глубине 4.5м и 3.5кВт насос дают мне в среднем 13кВт тепла. На мои 240м2 за глаза хватает, летом догреваю ГВС через косвенник.
310 462730
>>62710

>мгновенно взаимодействовали


Так это не взаимодействие
311 462735
>>62730
А что тогда? Сферическое растяжение волновой вероятностной функции и разрыв единой квантовой системы в вакууме?
312 462737
>>62735
Ну скажем так... мне нравится думать просто про многомировой вариант. Больно всё просто получается в этом случае с объяснением.
313 462741
>>62737
А может всё-такие есть лазейка типа внепространственной мгновенной связи какими то переносчиками типа тёмной материи? Что-то там с кротовыми норами размером с частицу?
314 462742
>>62741
В таком случае неизбежно нарушение принципа причинности, и связанные вместе с ним парадоксы.
315 462743
>>62737
Там кстати всё нифига не просто. Просто только на первый взгляд. Многомировая интерпретация открывает в 10 раз больше новых вопросов, чем отвечает на старые.
316 462744
>>62742
Хорошо не мгновенной, а много большей скорости света. Энштейнофаги скажут ересь, но почему и нет при ОПРЕДЕЛЁННЫХ масштабах около планковских? ВРУГ поле хигггса при этом будет незаметным. Нужно искать лазейки и совершенствовать инструменты.
317 462746
>>62744
А теперь понятно. Ты очередной шизик, который совсем не представляет о чем говорит. Поверхностно нахватался из энциклопедии разных терминов, и перерабатывает их в поток шизофазии тут на двачах. Съеби плес.
318 462747
>>62710
Это не взаимодействие. Ограничение по скорости тольк она взаимодейсвтие.
Все остальные скорости могут быть быстрее скорости света. Если фотоны запутаны, значит они уже провзаимодействовали в прошлом, а наше понимание запутанности такое не интуитивное, потому что квантовая механика для нас такая же не интуитивная.
Вот ты Вася, а твой брат близнец Петя. Я вас не могу различить, но как только ты говоришь что ты Вася, то я сразу знаю что твой брат Петя. Не похоже на взаимодействие да?
Имхо весь прикол запутанности возникает из-за нашего "неправильного" понимании времени.
319 462748
>>62735

>разрыв единой квантовой системы в вакууме


Вообще-то да.
Поэтому и называют коллапсом волновой функции.
Если смотреть по-хорошему, то измерение это взаимодействие квантовой системы с большой классической(которая тоже квантовая, но она достаточно сложная и мы отбросили все ее внутренние параметры, оставив среднее). Коллапс же функции выглядит мгновенно из-за то, что мы стыкуем "среднее" и целый ворох соответствующих ему величин.
Тут хорошая такая аналогия, вся "реальная" физика происходит на микроуровне, а на макроуровне мы видим только маленькую его часть. Из-за того что мы большие, мы не можем снизойти на микроуровень как микрообъект, поэтому мы обречены смотреть на микромир через "окна", которые показывают только осколки реальности. Мы конечно можем посмотреть на "голую" частицу, но из таких же частиц состоит наше же окно. И ради одной частицы нам надо что-то сделать с ними, теряя полноту описания системы.

Теперь о запутанности частиц. Запутанности частиц это тупо измерение некоторой системы через посредников. Причем посредники тут выступают и пара запутанных частиц, и пара измерителей/детекторов. Чтоб измерить запутанности нужно избегать нарушение когерентности частиц и детекторов, а это довольно сложная задача. И тут весь фокус в том, что условие соблюдение когерентности эквивалентно создание одного большого детектора, который пытается мерить не эти две частицы, а изначальную систему. Возвращаясь к нашей аналогии получаем, что мы смотрим через окошко, которого определенным образом(это важно) поделили на два и более, на тень "голой" частицы на смутном говне - пара запутанных частиц. Если расписать уравнения системы, то там выходит нагромождения операторов, которые сводится к одному. И никакой магии.
320 462749
>>62747

>Имхо весь прикол запутанности возникает из-за нашего "неправильного" понимании времени.


И какое же понимание времени по-твоему "правильное"?
И аналогия с близнецами кстати совсем некорректная. Близнецы изначально знают кто из них кто. А вот частицы не имеют определенного состояния. Их состояние определяется только в момент наблюдения. И вот когда мы пронаблюдали одну из частиц, её состояние определилось, и информация об этом состоянии мгновенно передается другой запутанной с ней частице. Для объяснения этого явления не обойтись без различных интерпретаций КМ. Тут либо миры разделились с проекцией разных состояний на каждый мир (и в таком случае да, взаимодействия не было). Либо какие то волны исходят в прошлое и будущее, которые несут в себе информацию не нарушая причинность и лимиты скорости (и в таком случае взаимодействие было). Либо мы постулируем еще какую нибудь неведомую хуйню (типа реальной конфигурации) в дополнительном пространстве.
321 462751
>>62749
Нет. Я думаю все проще. Квантмеху (физике частиц на микроуровне) похуй на пространство. Пространство валидно только для микросистем.
322 462752
>>62751

>пространство


Только для макросистемфикс
323 462753
>>62751

>Нет. Я думаю все проще


О да, твоё мнение для квантовой механики, вселенной и научного сообщества безусловно очень авторитетно.
324 462754
>>62749

>И какое же понимание времени по-твоему "правильное"


Да хуй знает. Просто мы мартышки которые борятся с собственной энтропией, я не ебу. Новая теория все поставит на места.

>Близнецы изначально знают кто из них кто. А вот частицы не имеют определенного состояния.


Аналогия кривая, да, но в общем она показывает, что мы сами не совсем понимаем, что такое запутанность. Мы не можем сказать, что частицы не имеют определенного состояния не измерив, поэтому мы и не знаем. Все опыты, типо квантового ластика или смена запутанности с задержкой не отвечает однозначно на вопрос: имеют ли частицы определенное состоние после запутанности. Без новой терии тут никак.
325 462755
>>62754

>Мы не можем сказать, что частицы не имеют определенного состояния не измерив, поэтому мы и не знаем.


Охх откуда же вы такие лезете рассуждать о таких сложных вещах, не зная даже элементарной матчасти, которую преподают в любом техническом вузе на 1-2 курсе физики? Почитай про неравенства Белла братан, почитай.
326 462756
>>62755

>Охх откуда же вы такие лезете рассуждать о таких сложных вещах, не зная даже элементарной матчасти.


Из технического вуза лезу.
Я не понимаю как то что ты тут написал опровергает это высказываение. Ты сам то понимаешь что неравенство Белла говорит или просто так сказанул?
327 462757
>>62755
Я вообще понимаю, как раз на 1-2 курсе физфака студент становится слишком самоуверенным в своих "знаниях", мол познал истину, а другие нет. Знаю, проходил этот этап.
328 462758
>>62756

>Мы не можем сказать, что частицы не имеют определенного состояния не измерив, поэтому мы и не знаем


Мы можем это сказать. Нарушение неравенств Белла прямо об этом говорит.
>>62757
Я читал про исследование, которое говорит об обратном. С ростом образования, у человека появляется большая неуверенность в собственных знаниях. Более образованные люди чаще склонны полагать, что окружающие имеют либо такой же уровень знаний, либо превосходящий.
329 462759
>>62758

>С ростом образования, у человека появляется большая неуверенность в собственных знаниях.


1-2 курс - это начало пути, если что, по этой логике должна быть максимальная уверенность, про что я и говорю как раз.

>Нарушение неравенств Белла прямо об этом говорит.


Давай тогда определи что такое измерение которое магическим образом награждает систему параметрами?

>Все опыты, типо квантового ластика или смена запутанности с задержкой не отвечает однозначно на вопрос: имеют ли частицы определенное состоние после запутанности. Без новой терии тут никак.


Ты видимо перепутал мою неопределенность и недовольство пробелами в теории с неким определенным отверждением.
330 462760
Вообще я, как физик, прекрасно живу с квантовомеханическими фактами. Только сверхсвет в "передаче/телепорте" информации слепым пятном в глазу сидит когда об этом вспоминаю.
331 462761
>>62760

>"передаче/телепорте" информации


>физик


Это анекдот какой-то?
332 462762
У меня вопрос и вообще не по теме раздела.
Нахуя нужны сурдопереводчики в телевизионных передачах? Глухие же могут просто прочитать субтитры. Зачем нанимать кого-то, чтоб он руками махал?
Это наверняка и сложнее понять, чем просто субтитры прочитать.
333 462763
>>62761
Ну для тебя разве что. Снежинка познавшая истину, расскажи какие слова мне следует, как физику, говорить.
334 462764
>>62763
Я просто не понимаю где ты нашел сверхсветовую передачу информации.
335 462765
>>62760
Значит как ты можешь быть спокоен? Значит ты все неправильно понял?
336 462766
>>62764
Речь о квантовой информации. Тебе макроскопическому не понять
337 462768
>>62760

>Только сверхсвет в "передаче/телепорте" информации


>Физик


Хуизик ты, вот кто.
338 462770
>>62766

> о квантовой информации


О, эта та самая. С помощью которой нельзя передать реальную.
339 462782
>>61534 (OP)
Как прослушивание музыки влияет на концентрацию и работу мозга? Мешает сконцентрироваться, помогает концентрироваться или вообще не влияет на концентрацию? И если в музыке звучат слова на понятном тебе языке? Типа если я в этот момент кодинг изучаю например. Пытался найти вменяемую инфу в инете, но там с первой до последней страницы пиздабольство уровня гороскопов и гомеопатии. Есть какие-то исследования на этот счет?
340 462788
Периодически тут бывают треды о компьютерном моделировании материи и т. п. У меня же вопрос проще - про CAE, симуляцию в САПР, вычислительную механику и т. п. Кто-нибудь знает на каком уровне сейчас находится эта сфера ? Симуляции действительно способны заменить испытания ? Чего не хватает для роста: мат. моделей, их релизаций в САПР или специалистов способных выполнить расчёт в САПР ? Или просто приборов требующих серьёзных испытаний(которые было бы выгодно симуляцией заменить) не так много проектируется и поэтому отрасль мала и рост не нужен ?
341 462789
>>62788
Профессионал FEM в треде (в смысле учился и зарабатываю этим). У всех вычислений подобного рода есть две проблемы: люди которые сидят за подобным софтом (достаточно проебать сетку и результаты сложных расчётов будут не совсем верными), и скорость расчёта (чтобы посчитать какую-то сложную и большую конструкцию на разные нагрузки можно ждать целую ночь). Хотя смотря что считать, обычные статические задачки в строительстве решаются довольно быстро. Динамика, трещины, усталость и прочее в авиации или автомобилестроении считается уже гораздо дольше.
342 462790
>>62748
Тут собственно нужно другой вопрос нужно задать: а зачем собственно этот механизм неопределённости нужен в реальности? Если копчёные квантовики уже доказали, что с инструментами всё норм, нельзя сделать совершеннее инструменты и преодолеть эту неопределённость. Так получается, что кроме математики у физиков то и нет другой координатной сетки понятий по которой будет понятно назначение квантовых эффектов. И логика тут другая, вероятностная. Такое ощущение, на одних и тех же частицах существуют несколько параллельных вселенных - такой механизм оптимизации.
343 462791
>>62789

>Профессионал FEM в треде


А что такое FEM ?

>учился и зарабатываю этим


А где этому учат у нас в стране ? 2,5 крупнейших вуза ? Эта работа считается научной деятельностью или есть предприятия где прям внедрено в производство ? У меня есть знакомые как программисты так и инженеры-конструкторы, но даже они не всегда понимают на слух что это за отрасль и в чём вообще суть. Вот ты когда учился, вам как говорили: что это перспективная работа и за вами будущее(Моим знакомым программистам примерно так говорили) ? Чего больше математики, программирования или всё уже хорошо
автоматизировано и надо модели в софт загружать и он сам посчитает большую часть готовыми средствами ?
344 462794
Наукачики, напомните как называется раздел в терии игр, посвященный данной теме - есть допустим социум, где способности каждого участника оценивают все участники, и общая оценка формируется на основе среднего значения (или очень близко), но при этом, если участник дает оценку другого участника, которая очень сильно отличается от средней, вес данного участника при оценке данной способности уменьшается, или даже его собственная оценка данной способности уменьшается в зависимсоти от значения отклонения. Вообщем как гуглить все, что связано с этой темой?
345 462798
>>62791
FEM это один из чисельних методов решения системы уравнений по сути. Включает в себя очень многое и даёт чисельные решения почти всех задач начиная со статики, заканчивая термодинамикой и электроникой. И используется чуть менее чем во всех CAE программах.

> А где этому учат у нас в стране ? 2,5 крупнейших вуза ?


А я и не в снг, лол.

>Эта работа считается научной деятельностью или есть предприятия где прям внедрено в производство ?


Есть и на потоке среди R&D в крупных концернах.

>У меня есть знакомые как программисты так и инженеры-конструкторы, но даже они не всегда понимают на слух что это за отрасль и в чём вообще суть.


Я на свою магистерскую накодил как раз один из способов решения. Нечто вроде этого

>Вот ты когда учился, вам как говорили: что это перспективная работа и за вами будущее(Моим знакомым программистам примерно так говорили) ?


Хехе.

>Чего больше математики, программирования или всё уже хорошо автоматизировано и надо модели в софт загружать и он сам посчитает большую часть готовыми средствами ?


Математики и физики больше всего. Сама специальность самая сложная область для изучения в университете после чистой математики целиком и физики частиц. Может слышал про сопромат, который отсеивает половину курса? Так вот это только первая маленькая ступень. Чисто теоретически ты можешь захуярить по туториалу из ютуба и посчитать себе например полку для инструментов, и она скорее всего будет держаться. Но если пизданется, то ничего страшного. В продакшне такой подход не работает, если боинг пизданется из-за таких расчётов, то будет очень плохо.
346 462799
>>62794

>есть допустим социум, где способности каждого участника оценивают все участники, и общая оценка формируется на основе среднего значения (или очень близко), но при этом, если участник дает оценку другого участника, которая очень сильно отличается от средней, вес данного участника при оценке данной способности уменьшается, или даже его собственная оценка данной способности уменьшается в зависимсоти от значения отклонения


Reputation system или reputation mechanism.
https://pdfs.semanticscholar.org/6e10/bc1128b3372adfb8daa7bc184435ecd4464a.pdf
347 462800
>>62798
Отклеилось https://youtu.be/lyTULzvHhXw
Вот похожую решалку на диплом я и кодил.
348 462802
>>62644
почему?

отдельно продаются nir сенсоры. т.е. попробовать самому собрать спектрограф.
вот видео спектрографа из веб камеры:
https://www.youtube.com/watch?v=MgogwcXUIoc
у линзы камеры нужно снять IR фильтр.
349 462805
>>61534 (OP)
Каковы последствия для климата при нахождении на планете одного единственного материка?.. Ведь если у нас лишь один громадный кусок суши, омываемый со всех сторон океаном, то должны быть какие-то ключевые последствия для климата на этом материке?
350 462808
>>61534 (OP)
Если взять бактериальную клетку, выкорчевать из неё нахуй ДНК и всю хуйну повинную в размножение, поместить клетку в среду без хищников, агрессивных воздействий, будет ле клетке требоваться пища/энергия или нет?
351 462810
1 Правильно ли я понял, что это соединения самых редких металлов на земле?
2 Как безопасно перевести осмий из оксида в металл?
3 Есть ли смысл закопать это и откопать через 10-20 лет?
4 Какой профит можно извлечь?
352 462811
>>62810
Самый редкий необязательно самый дорогой.
Цены прочекал?
353 462812
>>62808

нет, пища ей уже будет не нужна.
354 462813
>>62805

хороший вопрос!
предположим громадный кусок суши о котором ты говоришь равен общей площади суши на Земле, то есть 30% поверхности планеты, а остальное это океан. В этом случае последствия будут катастрофические, так как у нас появится огромное пространство, подобное южной части Тихого океана, где практически нет морских микроорганизмов. Морские микроорганизмы(вирусы, бактерии, археи, альджи) являются основными игроками в циклах углерода и азота на нашей планете. Если сократить их количество, то количество парниковых газов в атмосфере экспоненциально возрастет, что повлияет на климат в значительной степени.
355 462816
>>62813

>где практически нет морских микроорганизмов


Почему их нет?
356 462818
>>62805
Ну на большей части суши будет континентальный климат. В центре скорее всего будут пустыни в пустыне не обязательно жарко.
357 462821
в книгах при знакомстве с дифракционными решетками, свет направлен под прямым углом к решетке.
но в самодельных спектрографах, решетка расположена под углом к падающему на нее свету.
при этом, как бы, угол света за решеткой отклоняется на меньший угол, чем если свет под прямым углом к решетке и "полосу спектра" таким образом можно сфокусировать в небольшой объектив камеры. как это объяснить?
зная размеры объектива, нужно подобрать угол так, что бы весь спектр попал в объектив. делается методом тыка.

коробка куда устанавливается камера имеет щель и длину. к тому-же изнутри, коробка окрашена в черный цвет.
как я понял, это нужно для того, что бы лучи попадающие на решетку были сонаправлены, иначе выходной угол из решетки будет разным (см. первый параграф). ну а черный цвет нужен для поглощения лучей которые не со направленны.

дифракционная решетка получается из DVD диска.

ПО для отображения спектра есть под винду и под линукс. но под линукс с закрытыми исходниками. хуй знает почему не открыл, так-как он ПО не продает. прост нет исходников на сайте.
и нужно будет самому писать под линукс.

ну и нужны лампы для калибровки.

нужен контейнер который будет установлен за щелью и в которой можно положить вещество для анализа. хз пока, попробую вырезать стеклышки.

и не ясно какие ограничения по спектру у веб камеры.
0.jpg27 Кб, 480x360
358 462822
>>61534 (OP)
С некоторых пор в сети стало не так просто найти ВУЗовские учебники/задачи/решения. Гугл во всяком случае в этом не очень хорошо помогает. В мое время это было не так.

Ну, что это за пиздец? Это учебник?
http://files.lib.sfu-kras.ru/ebibl/umkd/353/u_course.pdf
Это что за пиздец?
http://cours.su/Tverdoe_telo/kinematika_tela.html
Это какой-то сборник заданий для курсовых работ, зачем он мне?
http://lib.agu.site/books/146/1/
Где ВУЗовский материал, динамика твердых сред, динамика деформируемых сред, все как у белых людей? Где задачи блеять? Кому-то нужно готовиться.
359 462823
Примерно месяц назад я это обнаружил. Сегодня все стало - да.
360 462826
Утверждение о том что электрон элементарная частица и неделим доказано эксперементально?

Если нет, то так считают потому что на это куча предпосылок и это предположение лежит в основе многих фундаментальных вещей?
361 462827
>>62822
Инженер, да?
362 462829
>>62826

>Утверждение о том что электрон элементарная частица


Это всего лиш утверждение - аксиома. В теории стрнуд другое утверждение.

>неделим доказано эксперементально?


В формализме кпт - да. Неделим и это доказано эксперементально. Ссылок кидать я, конечно же, не буду.
363 462830
Почему все химики такие тупые? Они принимают на веру всё, что им говорят физики, потом теория меняется, физики проводят шершавым им по губам, а сами химики с покерфейсом говорят "ладно".
364 462835
>>62811
Прочекал
Думаю заказать 2 кольца из него, может даже с осмием
365 462837
как вкатиться в инженерию , в ту где илон маск
366 462841
>>62830
Какая-то ложная дихотомия у тебя.
367 462847
>>62837
инженерия для отсталых, крутые поцики только в абстракции.
368 462850
Поясните за физику. Вот если я движусь с околосветовой скоростью без ускорения, окружающие объекты, которые пролетают мимо меня, будут иметь большую массу нежели если бы я находился в покое?
369 462852
>>61534 (OP)

На свет действует гравитация? На каждый вонючий фотончик?
370 462853
>>62850
нет
371 462854
>>61641

>фотон взаимодействует с магнитным полем



Подробнее плиз
372 462855
>>62852Ага, вижу википедию.

Какого хуя гравитационное отклонение света согласно ОТО больше ньютоновского в 2 раза?
373 462856
>>62854
Да пиздит он, не взаимодействует, фотон элемент группы U(1) которая описывает эм взаимодействие. А не он каким то образом взаимодействиет с этим полем. Хуйня все.
374 462859
>>62856
Может он имел ввиду фотон-фотон взаимодействие. Которое через позитрон-электронную пару рисуют в диаграммах фейнмана
375 462865
>>62853
Но почему? В моей системе отсчета все движется с околосветовой скоостью кроме меня, а обекты на таких скоростях увеличивают массу согласно формуле

>При скоростях близких к скорости света масса тела увеличивается, становясь бесконечно большой при v = c:

376 462867
>>62865

> а обекты на таких скоростях увеличивают массу согласно формуле


Тебя сразу уринировать?
377 462869
>>62867
Ты можешь конкретно по теме пояснить, а не хвастаться знанием туалетных терминов?
378 462871
>>62869
Масса инвариант независящий от скорости.
380 462873
>>62872
Обсуждалось тонну раз, на всевозможных форумах, даже тут на сосаке. Ладно еще раз, читаем тут
https://mipt.ru/education/chair/physics/S_I/method/Okun.pdf
381 462874
>>62872
Никто не использует понятие релятивистской массы, так как масса такая теряет свой смысл массы.
382 462875
>>62874
Ну вот в коллайдере электрон не могут разогнать быстрее ~99% от с потому что масса увеличивается
383 462876
>>62875
Ты понимаешь что такой электрон по движению "сопротивляется" сильнее чем перпендикулярно движению? Ты эту массу релятивистскую, даже никуда прикрутить не можешь, чтобы она там как масса была. Пользуйся энергией E=E0/sqrt(1-v2/c2)
384 462877
>>62876
Но масса релятивистская есть же?
385 462878
>>62877
Нету, это неудачный термин.
386 462879
>>62878
Ну электрон же сопротивляется, тогда почему в релятивистской формуле это сопротивление записывается как масса, а не например как дополнение к массе в виде меры сопротивления движению тела имеющего массу покоя? Зачем осознанно отождествлять что-то похожее на изменение массы и настоящую массу тела?
image.png123 Кб, 637x574
387 462880
>>62879

> в релятивистской формуле это сопротивление записывается как масса


В старых учебниках. Даже в ЛЛ уже об этом пишут, и мол "хуйня а не термин".
388 462881
>>62879
Читай >>62873
На этом этот вопрос закрыт. Всё остальное это уже троллинг(тупостью или нет уж не знаю)
389 462905
Чисто дилетантский вопрос. Вот я качаю мышцу, когда хорошо сделал - онаболит потом. Это значит растёт. Так понимаю в ней новые капилляры прорастают. А в мозге можно стимулировать рост новых сосудов ?
390 462907
Вот есть объект в межпланетном пространстве солнечной системы, если придать ему небольшое ускорение он же будет лететь прямо пока какая нибудь планета не притянет его к себе? и может ли он покинуть пределы солнечной системы?
И вообще когда запускают всякие зонды на марс все топливо тратится нп подъем ракеты в космос и торможение потом при подлете? или в процессе полета оно тоже затрачивается на совершение каких-то маневров? где можно найти маршрут полета какого то объекта с земли на марс?
391 462909
>>61534 (OP)
Я прошу прощения, в медаче 1.5 анона, я немного переживаю. Нужна помощь химиков.
Сегодня в лаборатории разлил какую-то жижу, с запахом ссанины. От паров щипало глаза и нос.
Это мог быть раствор аммиака? Или что-то другое? Я быстро убрал все это дело, промыл глаза. Делал все быстро общее время "щипания" составило где-то 5-7 сек. Чувствую себя норм вроде. Я буду в порядке?
сори за оффтоп
392 462919
>>62855
Ответьте, пожалуйста
393 462921
>>62907В космосе всякие зонды поворачиваются гироскопами

Да, будет лететь прямо и покинет солнечную систему
394 462922
>>62876Ну так поясняй за энергию, недалекий наш, какая будет энергия у объектов вокруг меня, если я двигаюсь со световой скоростью? Одинаковая? Шо у земли, шо у луны? А у чёрной дыры? У чёрной дыры вторая космическая скорость выше световой, она от меня как бы убегает, сечёшь? У неё отрицательная скорость. Че по энергии чёрной дыры тогда?
395 462929
>>62922
Я другой анон вообще. Наезжал на тебя какой-то злой студент.

> какая будет энергия у объектов вокруг меня


Большая. Так как в твоей системе отсчета вектор энергии-импульса у этих объектов повернут.

> У неё отрицательная скорость.


Это не очень важно, энергия не зависит от направления скорости. Математически выражается тем, что от квадрата скорости зависит.
396 462937
Почему большинство хищников не падальщики по совместительству? Это так трудно было сделать? И если у трупоедов есть обоняние, они не чувствуют вони?
397 462940
>>62909
Всё с тобой нормально будет, амин небось какой-нибудь разлил
398 462941
>>62937
Да, трудно сделать, трупный яд, вся хуйня
Запах отночителен. Для тебя воняет труп, только потому, что если ты его съешь - ты отравишься. Для падальщиков он пахнет вкусно
399 462942
>>62905
Мышца болит после тренировки потому что ты порвал часть волокон. Грубо говоря, когда ты надрываешь небольшую часть волокон, нарастает чуть-чуть больше. С мозгом это не прокатит, как ты понимаешь
тех.png21 Кб, 1188x840
400 462945
Будет работать?
401 462946
>>62922

>высер говна


Что сказать то хотел?
402 462947
403 462949
помоги с NIR спектроскопом.

1. не нашел фотодиоды для длин волны 700 - 2500 nm.
они до 1700 nm. пик.
чем измеряется интенсивность с длиной волны более 1700 nm?

2. как диодом считать спектр от дифракционной решетки? первое, что пришло в голову, это двигать диод. зная позицию диода и интенсивность, строится спектрограмма.
двигать моторчиком диод как в струйном принтере движется головка. или же вращать дифракционную решетку, что легче.

получается не дорого, разве что диод дорогой 50+ долларов.
404 462953
>>62949
1. нашел InGaAs детекторы 800-2600 nm, но цену производитель не указывает. на ebay - 180 долларов.. для меня дорого.
405 462963
>>62949
Можешь полазить по помойкам химфаков/физфаков мб что-нибудь целое найдешь.
406 462964
>>62929
Че? Энергия не зависит от вектора скорости? Ты что ебнулся?
407 462965
>>62946Выбрал говна тебе на ебало, ты ж не слепой
408 462966
>>62964
А это троллинг тупостью, понятно.
409 462973
>>62966
Бля, мудила, какой энергией будут обладать в отношении друг друга летящие друг за другом объекты по одной траектории?
notfound.jpg45 Кб, 500x500
410 462977
Успокойтесь
411 462979
>>62966

> не зависит от направления скорости

412 462981
Без названия.png274 Кб, 822x273
413 462992
О, лучше сюда, наверное.

Почему пикрил невозможен?
Понятное дело, что можно записать всякие законы сохранения импулься (с учётом импульса поля) и т.п.
Но наверняка есть какое-то элементарное качественно объяснение.
414 462993
>>62979
Ты дебил
415 462994
>>62992

Хз, построй с игрушечными машинками такое
416 462995
>>62992
Так на каждое действие, есть противодействие, с какой силой магнит притягивает машинку, с такой же силой машинка магнит.
417 462996
>>62993
Пхех, мда.
Мочух кстати тут, банить когда залетных траллей будут?
418 462997
>>62992
Ты же сам уже написал элементарное объяснение. Что может быть элементарней чем законы сохранения?
419 462998
>>62996
Но ты дебил же, посмотри на себя, жалкий цвырь
420 462999
>>62995Че ты несёшь
421 463000
>>62997
Это математическое обоснование принципов природы. Думаю, есть объяснение, которое ближе к бытовому.
>>62995
Ну, машина у нас условно говоря свободна в пространстве, а балка закреплена, там сила типа как компенсируется реакциями балки.
А вообще да, 3 закон вполне себе интегральный, он на всю систему тоже действует. Так что эти самые силы реакции, которые не дают магниту двигаться, передадутся через саму балку машине и будут ей не давать двигаться.
Вот это я затупил, мда.
2 курс лучшего физического вуза страны
422 463005
>>62999

>Че ты несёшь


Это жертва школьного образования. Когда детям на веру вдалбливают костыльную ньютоновскую механику, в которой твёрдая опора СООБЩАЕТ СИЛУ.
423 463006
>>63005
Я в ахуе
424 463007
Вот вам тупой вопрос: что быстрее притянется к магниту массой в 1 кг с расстояния 10 метров в вакууме - магнит массой 1гр или 10 грамм?
425 463008
>>63007
Ну типа твой магнетар полетит навстречу десятиграмовой гирьки
426 463009
>>63008
Почему?
427 463010
>>63007
На таком растоянии ниче не притянется хоть в вакууме хоть нет, если это не электромагнит
428 463012
>>63005
С каких это пор у нас 3 ЗН не выполняется в нерелятивистском пределе?
429 463013
Если разместить магнит над дорожкой из сверхпроводников в вакууме и разогнать его, то будет ли он двигаться дальше?
430 463016
Лол
431 463020
>>61974
Что за нахуй?
1328117625838.png31 Кб, 508x376
432 463021
>>61534 (OP)
1. Луна по баллистической траектории падает на Землю, просто очень долго.
2. Луна отдаляется, наплевав на пункт 1.
433 463022
>>63000

>лучшего физического вуза страны


Физтех надеюсь?
434 463024
>>63021
Вектор скорости луны будет постоянно направлен по касательной к её орбите. В идеале он уравновешивается притяжением. Но луна находится чуть дальше, чем нужно, чтоб было равновесие, из-за этого равновесие немного смещено и она по чуть-чуть отдаляется
435 463030
>>62949
не знаю если кому интересно.
обзор фотодиодов: http://mmrc.caltech.edu/FTIR/Literature/General/Hamamatus IR dectors.pdf
есть советские германиевые фотодиоды для длин волн 0.5 - 1.9 мкм. название: ФД-2. 2-3 доллара за штуку. пока хз какие ограничения анализа накладывает урезанный диапазон.

есть несколько публикаций конструкций NIR спетроскопов: https://www.mdpi.com/1424-8220/17/8/1914/pdf
пик: диф. решетка. не понял почему MEMS, кусок CD диска не подойдет? может MEMS легче и частота колебаний получается выше, чем с более массивной решеткой.
там же про восстановление спектра.
пишут, что для длин волн выше 1.8 um нужно охлаждение.
436 463035
существует ли способ аналитического решения систем вида
y=ax^2+bx
y=c*z^2+d
где a,b,c,d числа
437 463036
>>63035
Два уравнения, 3 неизвестных, решений бесконечно много.
438 463037
>>63036
а если необходимо найти только целые решения?
439 463039
>>63037
Ну тогда читай про диофантовы уравнения. Это весьма обособленный раздел математики. Спроси на math.
440 463043
Вопрос возможно не для этой доски, просто вдоховлён тредами отсюда. Тут где-то ещё висит антинаучная фигня про утраченные технологии строительства и архитектуры. От нечего делать я посмотрел видосы на ютуб где тонко намекали на то что Исакиевский собор и вообще Санкт-Петербург построен пришельцами/атлантами. Однако я действительно задумался о качестве и сложности постройки такого рода зданий около 200 лет назад и современном строительстве. У меня есть ряд своих примеров. Мой отец в 90-ые работал обычным строителем. Однажды он выполнял заказ в частном дореволюцеонном купеческом деревянном доме в исторической части города. В общем они разбирали полы и обнаружили под ними дубовые балки 100+ лет совершенно не подверженные гниению т. п. Это в отсутствии какой-либо химической промышленности. Единственное объяснение это проектировка фундамента с учётом термических характеристик дома(например в центре дома была русская печь, т. е. балки всегда оставались сухими, в них просто не попадала плесень и у неё не было условий для развития). Современные частные деревянные дома столько не простоят как этот. В общем я не могу понять почему известные технологии не получили широкого распространения. Я понимаю что после революции население стало рости быстрее, требовалась урбанизация чтобы рабочие жили ближе кфабрикам и т. п.. Из экономических причин невозможно было строить всем жильё такого качества. Но вот сейчас что мешает начать строить качественно жилые дома. Квартиры многоэтажных новостроек тоже не очень. Окна делаются вырывным методом(кран вырывает рамку из стены делая окно, стена искривляется), стены кривые. Во многих девятиэтажках есть трещина в одном итом же месте на потолках гостиной 3-ёх комнатных квартир. Почему знания о строительстве не получают какой-то широкой адапатации, не разрабатывается методик и инструментов для рабочих позволяющих выполнять обычный жилой дом в 21 веке на уровне хотя бы 70 % качества культового(или просто хорошо оплаченного в случае частных домов) сооружения 19 века ?
440 463043
Вопрос возможно не для этой доски, просто вдоховлён тредами отсюда. Тут где-то ещё висит антинаучная фигня про утраченные технологии строительства и архитектуры. От нечего делать я посмотрел видосы на ютуб где тонко намекали на то что Исакиевский собор и вообще Санкт-Петербург построен пришельцами/атлантами. Однако я действительно задумался о качестве и сложности постройки такого рода зданий около 200 лет назад и современном строительстве. У меня есть ряд своих примеров. Мой отец в 90-ые работал обычным строителем. Однажды он выполнял заказ в частном дореволюцеонном купеческом деревянном доме в исторической части города. В общем они разбирали полы и обнаружили под ними дубовые балки 100+ лет совершенно не подверженные гниению т. п. Это в отсутствии какой-либо химической промышленности. Единственное объяснение это проектировка фундамента с учётом термических характеристик дома(например в центре дома была русская печь, т. е. балки всегда оставались сухими, в них просто не попадала плесень и у неё не было условий для развития). Современные частные деревянные дома столько не простоят как этот. В общем я не могу понять почему известные технологии не получили широкого распространения. Я понимаю что после революции население стало рости быстрее, требовалась урбанизация чтобы рабочие жили ближе кфабрикам и т. п.. Из экономических причин невозможно было строить всем жильё такого качества. Но вот сейчас что мешает начать строить качественно жилые дома. Квартиры многоэтажных новостроек тоже не очень. Окна делаются вырывным методом(кран вырывает рамку из стены делая окно, стена искривляется), стены кривые. Во многих девятиэтажках есть трещина в одном итом же месте на потолках гостиной 3-ёх комнатных квартир. Почему знания о строительстве не получают какой-то широкой адапатации, не разрабатывается методик и инструментов для рабочих позволяющих выполнять обычный жилой дом в 21 веке на уровне хотя бы 70 % качества культового(или просто хорошо оплаченного в случае частных домов) сооружения 19 века ?
441 463044
>>63043

>Но вот сейчас что мешает начать строить качественно жилые дома.


Стремление паразитических пидарасов к максимальной прибыли с минимальной площади земли и с минимального количества купленных метров.
442 463045
>>63043
Нет потребности
443 463046
>>63044

>Стремление паразитических пидарасов


Это ты о рациональном мышлении? Оно, конечно, паразитическая сущность, но ориентацию я бы ей так запросто не приписывал. Хотя, большинство учёных мужчины...
444 463047
>>63046
Нет промытка или агитшлюха, не об этом.
445 463048
>>63047
Объясни мне, пожалуйста, как ты видишь существование науки и учёных в мире вне капитализма.

Учёные - это своеобразный мозг социума. Они жрут 60% ресурсов и нихуя ощутимого взамен дать не могут, кроме обещаний "через 300 лет наши правнуки победят рак". Чтобы финансировать этот мозг, нужно насильно отбирать у социума его деньги, потому что сам социум не тупой и кормить зажравшихся нахлебников (а далеко не все "люди науки" действительно относятся к науке) не собирается.

Для насильного отбора благ существует только капиталистическая машина. Другие показали свою нежизнеспособность. Так вот объясни мне, как будут работать твои Лысенки с Мичуриными, если климат будет подходящий.
446 463049
>>63048

>вне капитализма.


Проблема в том что сейчас это вне и есть.

>Учёные - это своеобразный мозг социума. Они жрут 60% ресурсов


Мозг который ничего не решает и сидит под шконкой? Напомни мне какое там процент бюджета на науку идёт...
Ну точно агит шлюха, пшол вон отсюда.
447 463051
>>63049

>Мозг который ничего не решает и сидит под шконкой?


Совершенно верно. Как и тот, который у тебя в голове. Все твои решения принимают железы, а мозг занимается их оправданием в бюджете.

>Напомни мне какое там процент бюджета на науку идёт


Говоришь с коммиблядком о капиталистическом финансировании - он с тобой о государственных дотациях. Ёбни себя лопатой, товарищ.
448 463052
>>63051
Стопэ. Но ведь в твоих "капиталистических машинах" фундаментальную науку финансирует тоже государство, "насильно отбирая у социума его деньги", никак не капиталовладельцы.
мимо
sage 449 463053
>>63049

>какое там процент бюджета на науку идёт


>имплаинг оборонка это не наука

450 463055
>>63049
На Западе почти всю научную деятельность финансировали частные лица, начиная с медичей и Ко. Причем зачастую правители и общество а целом хейтли ученных мужей. Всякие Екатерины и Наполеоны были исключением. И ничего, наука как двигалась и совершались открытия, которые общество долго переживало.
Особенная ситуация сложилась в Англии, где большинство ученных мужей были или лорды, или понравившимся им люди. Даже до сих пор за любое исследованные кто-то должен платить. Впрочем за века развивалась система грантов, фондов и стипендий, которые содержат не государство, а различные коммерческие или не очень организации. США унаследовали эту систему. И почему то никто не сомневаются в успехах их фундаментальной науки.
451 463058
>>63055
Огромные деньги на науку в США выделяются именно из гос.бюджета, разве что в биомедицинской отрасли частные инвестиции где-то половину от финансирования составляют.
452 463060
Я мальчик. У меня есть X хромосома от мамы. Она в свою очередь получила две x-хромосомы - от моих деда и бабки.
Та x-хромосома, что у меня - она конкретно от деда/конкретно от бабки ИЛИ какая-то их рекомбинация?
453 463062
>>63010
Не важно расстояние, идиот, суть вопроса ты понял?
454 463069
>>63060
Рекомбинация
455 463078
>>61535
Потому что
456 463085
>>63055
Пока физик пытался что-то пропихнуть в своей гейропке, уговорить инвесторов дать денек на его хуйню, математики в рашке и германике ебашили матешу, что аж на несколько веков вперед хватило.
457 463091
Почему не проводят тот эксперимент с обезьянами "у нас так принято"? Охуительно же будет, если он не подтвердится.
458 463092
Как узнать мощность в ваттах на квадратный метр для видимого света яркостью 80000 люкс?
459 463093
Насколько мощнее будет атомный взрыв, если удастся удержать цепную реакцию до полной выработки соотв. количества плутония?
460 463095
>>63085
Ну для развития матеши много средств не надо. Обугленная мочалка да спина товарища по бараку - вот тебе и письменный стол.
461 463098
>>63091
Что за эксперимент? Опиши вкратце
462 463099
>>63098

> Клетка. В ней 5 обезьян. К потолку подвязана связка бананов. Под ними лестница.


> Проголодавшись, одна из обезьян подошла к лестнице с явными намерениями достать банан. Как только она дотронулась до лестницы, вы открываете кран и из шланга поливаете ВСЕХ обезьян очень холодной водой. Проходит немного времени, и другая обезьяна пытается полакомитЬся бананом. Те же действия с вашей стороны. Третья обезьяна, одурев от голода, пытается достать банан, но остальные хватают ее, не желая холодного душа.


>


> А теперь уберите одну обезьяну из клетки и замените ее новой обезьяной.


> Она сразу же, заметив бананы, пытается их достать. К своему ужасу, она увидела злые морды остальных обезьян, атакующих ее.


> После третьей попытки она поняла, что достать банан ей не удастся. Теперь уберите из клетки еще одну из первоначальных пяти обезьян и запустите туда новенькую. Как только она попыталась достать банан, все обезьяны дружно атаковали ее, причем та, которую заменили первой еще и с энтузиазмом.


> И так, постепенно заменяя всех обезьян, вы придете к ситуации, когда в клетке окажутся 5 обезьян, которых водой вообще не поливали, но которые не позволят никому достать банан.

463 463100
>>63099
И в чем суть то? Что такого прорывного в этом?
15533680329130.png462 Кб, 700x831
464 463107
>>63100
Суть в том, что анончик считает, как будто эксперимент выдуман и никогда не ставился. Он думает, что если будет обнародована блистательная истина "человек = трусливое стайное молокососущее", то мир встанет на уши. Анончик долбоёб и не знает, что наряду с трусостью, глупостью и ленью мы развивали ещё и рационализацию, компенсацию и вытеснение, позволяющие нам игнорировать абсолютно любые факты с простым ебалом.
465 463112
>>63107

>игнорировать


Это не оче, гораздо лучше начистить ебасос кукарекающему ананчику, типа этого - >>63099
На улицу-то если и выходит то глазки прячет и тихо ведет себя этот червь-питурд, так-то.
466 463122
Так мужики, объясните тупому перваку, как это, что фотон не имеет массы покоя? Откуда же тогда масса?
467 463125
>>63107
>>63112
Пиздец вы злые, я вообще никакого умысла не имел, простой вопрос задал. Лучше думайте, что с вами не так, любого хуесосить бросаетесь. Хорошо вам, привыкли тут сраться, на всё ответ есть. А я миролюбивый, не знаю, что ответить даже.
468 463126
>>63107

Хуй знает, откуда и выводы такие:

> Он думает, что если будет обнародована блистательная истина "человек = трусливое стайное молокососущее", то мир встанет на уши.


нет, я этого не думал, а вот кто так думает, ссылается на эту пасту как документированный результат.
469 463141
>>63122

>Откуда же тогда масса?


Ты про какую? Они разные бывают.
470 463142
>>63122
Тензоры проходил?
Функцию Лагранжа проходил?
Если нет, то иди нахуй в армию. Вот тебе ликбез. Тензор это объект из линейного пространства, который по своей сути не меняются, меняются только его разложение по координатам в различных базисах. Тот же вектор, он не меняется если его разложит в двух различных базисах, но вот координаты да. У каждого тензора можно найти некоторую не изменяемую величину - инвариант.

Теперь про функцию Лагранжа Каждый возможной траектории приписывается число - действие. Эволюции физический систем идут так, что из начальной точки выбирается только одна траектория, где действие наименьшее. Для отыскание этой траектории вводится функции Лагранжа системы или лагранжиан. Грубо говоря лагранжиан выглядит как дифура, а его решение является нужными кривыми. И если мы возьмем совсем простой и обобщенный случай, то мы получит старые добрые законы Ньютона.
Введение лагранжиана кажется чем-то излишним, есть же законы ньютона. Однако из лагранжиан естественно выводятся интегралы движения, т.е получаются дифуры, которые еще легче решать. И одним из интегралов движения является энергия и импульс.

Теперь Теория относительности, тут тоже самое только вот у нас пространство 4-мерное, да с еще с особым измерением расстояний между точек, ну еще вводится понятие собственного времени или путь по кривой в 4 пространстве. Для систем составляем лагранжиан, и ищем интегралы движений. а потом обнаруживаем, что интеграл движений представлен 4-тензором, в компоненты которого входят энергия и импульс, а инвариант этого тензора масса. И если мы возьмем тензор с нулевым инвариантом, то он вполне легален для теории. Так из теории относительности можно вывести, что существуют безмассовые частицы.
471 463143
Так скоростное замедление времени чисто геометрическое?
Потому что свету скорости взаимодействия требуется больше времени чтобы пройти большее расстояние провзаимодействовать на бОльшем расстоянии, увеличивающемуся из-за движения?
472 463145
>>63143
Ты перепутал.

>требуется больше времени


Меньше

>на бОльшем расстоянии


меньшьем
473 463146
>>63142
ты токсчиный, будто не на двачах сижу, а хуй знает где
474 463147
IMG20190326153226.png51 Кб, 720x879
475 463148
>>63145
С чего бы это?
Происходит замедление, а не ускорение.
123143523.jpg28 Кб, 596x556
476 463149
>>63148
Потому что так работает гепирболические преобразования и теория относительности в целом.
477 463150
>>63142
Да, спасибо.
тензоры ещё не проходил, я биолог, просто физика у нас полноценная, а матан с какого-то хуя нет
478 463151
>>63149
Непонел...
Можно проще, что не так с моим рисунком?
Если скорость взаимодействия константа, а расстояния между частицами которые должны провзаимодействовать больше, следовательно на одно условное взаимодействие будет уходить больше времени, т.е. оно, условное, относительное время для тела, состоящего из этих частиц, будет замедлено.

Конретно касательно рисунка, в каком месте с примером на нём что-то не верно?
479 463157
>>63151
Он тебя запутал.
В теории относительности вводиться пространство-время, где расстояние меряются другими способами.
Упрощенный пример
s^2=(c(t-t0))^2 - (x-x0)^2
Ну если мы будем смотреть через дифференциалы, то ds^2=c^2dt^2 - dx^2. Поскольку dx=v(t)dt, то получим ds^2=dt^2(c^2-v^2). Чтобы перейти в систему отсчета связанной с движением, нам надо совершить линейное преобразование, которое сохраняет соотношению расстояний. Такое преобразование есть - гиперболический поворот:
t' = t ch a + y sh a
x' = t sh a + x ch a
где a = v/c.
Лень расписывать все, но суть в том, что в новой системе отсчета t-сократилось. Время относительно. Чтобы как-то увязать физические процессы вводят понятие собственного времени. Его суть проста, тело движется по некоторой кривой, длина пройденного пути в любых системах отсчета одинаковая. С самим путем работать не удобно, его линейно параметризуют некоторым параметром, которое и называют собственным временем. Относительно собственного времени считают физическое действие.

Из-за особенности измерения расстояний, выбирается самый длинный по абсолютной величине путь, но тупо умножению на -1, самый длинный превращается в самый короткий.
480 463160
>>63149
Зай, я не понимаю, как может замедляться время, когда ты в уме это время можешь считать с каким угодно ритмом. Вы ебнутые все
481 463161
>>63151
Да, все правильно, просто в физике много идиотов, и они готовы считать за замедление времени переход воды из текучего состояния в лёд. Хули, ниче не движется же, значит и времени больше нет.
482 463162
>>63157Я же сказала, вы ебнутые идиоты
483 463165
>>63157
Если за время взять it. То можно остаться в обычных поворотах на синусы и косинусы.
484 463166
>>63165
во первых нет, во вторых удачи перейти в комплексную плоскость. ты даже через расложение по экспонентам не получишь тот же результат.
485 463168
>>63166

> во первых нет


Надо просто не забыть, что sh(ix)=isin(x)
а ch(ix)=cos(x)

> удачи перейти в комплексную плоскость


Вообще не вижу проблемы. Метрика зато симметричная, удобно брать дифференциалы.
486 463169
>>63168
Не проще в тензорах?
487 463170
>>63160
>>63162
Оп, посудомойка порвалась
488 463175
>>63168
не понимаю как ты простой заменой t на it переходишь в синус и косинус.
489 463184
Блен, а если вселенная расширяется, и уже пиздец как расширилась от начала...
То что, плотность флуктуаций падает? Да не, тогда учитывая пиздецовые порядки расширения уже случившегося и продолжающегося материя вряд ли смогла бы существовать в том виде, в котором и была раньше, а насколько известно вроде, она никак не поменялась по крайней мере начиная со времени 400т. лет после большого взрыва.

А если плотность флуктуаций не падает, то просто сам факт расширения вселенной пораждает дополнительную материю.
А хотя не, не порождает, она же появляется и исчезает.
Но чтобы она появлялась и исчезала с одинаковой плотностью и частототой на всё большем пространстве, то должна расти "полная мощность" всего поля, суммарная мощность, требуемая для такого процесса порождения и аннигиляций частиц. Хотя в данном случае будет правильнее сказать энергия поля.
Энергия поля растёт по фатку его расширения.

Ухх бля, пиздец...
490 463186
>>63175
-s^2=(сit)^2+x^2+y^2+z^2
Это почти евклидовая метрика, поворот в плоскости x и t можно делать как на обычной плоскости.
x`=cos(a)x+sin(a)(cit)
t`=sin(a)x+cos(a)(cit)
Интервал сохраняется, проверь сам.
491 463187
>>63184
А ты как представляешь расширение вселенной?
Мне кажется, если смотреть твоим взглядом, то флуктации это самая маленькая проблема - у тебя все константы пиздой пойдут. Это вообще будет совершенно другая физика.
Вот у тебя вселенная "расширяется" - а флуктации, энергия, материя - нет? Они из другой вселенной?
492 463191
>>63186
Проебал минус в первой формуле. Но суть ясна.
493 463193
>>63191
>>63186
Ну и да, угол поворота очень красиво получается:
a=arctg(iv/c)
494 463205
>>63170
Спермобак аргументирует
495 463206
>>63184
Чем больше она расширяется, тем больше вещества оттягивается от центра к периферии. В конце концов все остынет, а потом стянется обратно и опять сожмется в одну точку. А потом опять взорвется
496 463207
>>63206
Почему сожмется? Притяжение же убывает с расстоянием
497 463208
>>63205
Ну бля, а как это по другому назвать то, если не подрыв
498 463209
>>63206
Вопрос и мозгосрыв не в этом
499 463210
>>63187
У тебя эта часть твоего поста:

>Мне кажется, если смотреть твоим взглядом, то флуктации это самая маленькая проблема - у тебя все константы пиздой пойдут. Это вообще будет совершенно другая физика.


Противоречит этой части:

>Вот у тебя вселенная "расширяется" - а флуктации, энергия, материя - нет? Они из другой вселенной?

1553054806-0038200285a4f6c35f89154242acf693.jpeg39 Кб, 700x420
500 463212
как научно обосновывается явление внимание в контексте термодинамики?
например у нас есть 2 жилых и одновременно рабочих помещения.
в одном бегноним слепо-глухо-немой без рук и ног, который не может использовать своё внимание как вектор для наведения порядка в помещении.
в другом здоровый человек который просто предоставлен сам себе.
в обоих помещениях есть хавка, плита, какие-то книги - но всё дико разбросано и очень грязно.
мы берём и, исключив из рассмотрения человеков, измеряем энтропию помещения. считаем все тупо физической системой и для простоты считаем обычную энтропию ну или какую-то монотонную функцию от неё, назвав её "амплитудой упорядоченности".
дальше запускаем эксперимент.
оба человека предоставлены сами себе, у них одинаковый запас кислорода, воды, электричеств.
один, если он нормальный человек, в первую очередь наведёт порядок в помещении - уберёт грязь, сложив её аккуратно в мусорное ведро, упорядочит предметы, поставив их на полки и так далее.
второй нихуя делать не будет - только жрать, спать, дышать, срать и может быть поползает по помещению, исследуя его с помощью тактильного ощущения на коже. при том он, скажем, не способен двигать кистями особо и следовательно навести порядок. зато ему нечем себя занять, он много ползает, играется как-то в аутичном стиле
https://www.youtube.com/watch?v=4P79Y-lJz1E только еще, скажем, менее трудоспособный.
оба тратят равное кол-во каллорий и теплововыделение у них одинаковое.
проходит время, их запасы кончаются, мы их выводим из помещения и начинаем измерять энтропию вновь.

в "чистой" комнате - где убрана пыль, всё вымыто, частицы грязи и говна собраны в отдельном объеме мусорного ведра, предметы расставлены в положения с потенциальной энергией выше изначального (книги убраны с пола в шкаф).
в грязной комнате - стало еще больше беспорядка.
в первом случае в системе находился организм, реализующий явление произвольного внимания, который можно представить, как выделенный в пространстве конус,как луч прожектора, направляемый организмом на физическую систему и оказывающий действие.
во втором случае такого "конуса" не было, хотя, предположим, оба организма генетически идентичны, но их фенотип отличается детализацией сенсорной активности и обменом информации мозга с окружающей средой.

так вот, наличие произвольного внимания по идее вносит в систему нечто особенное, в результате чего энтропия системы нарастает медленнее. возникает вопрос о термодинамике произвольного внимания.
однако гугл по запросу "термодинамика внимания", энтропия внимания - нихрена не выдает.
чё каво а?
1553054806-0038200285a4f6c35f89154242acf693.jpeg39 Кб, 700x420
500 463212
как научно обосновывается явление внимание в контексте термодинамики?
например у нас есть 2 жилых и одновременно рабочих помещения.
в одном бегноним слепо-глухо-немой без рук и ног, который не может использовать своё внимание как вектор для наведения порядка в помещении.
в другом здоровый человек который просто предоставлен сам себе.
в обоих помещениях есть хавка, плита, какие-то книги - но всё дико разбросано и очень грязно.
мы берём и, исключив из рассмотрения человеков, измеряем энтропию помещения. считаем все тупо физической системой и для простоты считаем обычную энтропию ну или какую-то монотонную функцию от неё, назвав её "амплитудой упорядоченности".
дальше запускаем эксперимент.
оба человека предоставлены сами себе, у них одинаковый запас кислорода, воды, электричеств.
один, если он нормальный человек, в первую очередь наведёт порядок в помещении - уберёт грязь, сложив её аккуратно в мусорное ведро, упорядочит предметы, поставив их на полки и так далее.
второй нихуя делать не будет - только жрать, спать, дышать, срать и может быть поползает по помещению, исследуя его с помощью тактильного ощущения на коже. при том он, скажем, не способен двигать кистями особо и следовательно навести порядок. зато ему нечем себя занять, он много ползает, играется как-то в аутичном стиле
https://www.youtube.com/watch?v=4P79Y-lJz1E только еще, скажем, менее трудоспособный.
оба тратят равное кол-во каллорий и теплововыделение у них одинаковое.
проходит время, их запасы кончаются, мы их выводим из помещения и начинаем измерять энтропию вновь.

в "чистой" комнате - где убрана пыль, всё вымыто, частицы грязи и говна собраны в отдельном объеме мусорного ведра, предметы расставлены в положения с потенциальной энергией выше изначального (книги убраны с пола в шкаф).
в грязной комнате - стало еще больше беспорядка.
в первом случае в системе находился организм, реализующий явление произвольного внимания, который можно представить, как выделенный в пространстве конус,как луч прожектора, направляемый организмом на физическую систему и оказывающий действие.
во втором случае такого "конуса" не было, хотя, предположим, оба организма генетически идентичны, но их фенотип отличается детализацией сенсорной активности и обменом информации мозга с окружающей средой.

так вот, наличие произвольного внимания по идее вносит в систему нечто особенное, в результате чего энтропия системы нарастает медленнее. возникает вопрос о термодинамике произвольного внимания.
однако гугл по запросу "термодинамика внимания", энтропия внимания - нихрена не выдает.
чё каво а?
501 463217
Правильно ли понимаю что в процессе туннельного эффекта конкретный электрон никуда не перескакивает, а поглощается барьером с одной стороны и соответственно выбивает электрон с противоположной?
502 463218
>>63217

> конкретный электрон


В квантовой механике добрая половина вещей завязана на том, что конкретных электронов нет.
503 463219
>>63218
Всё время забываю, надо спрашивать какая вероятность что ... То есть вероятность уже как минимум 0,5 что ж неплохо.
504 463221
>>63219
Просто туннелирует же волна. Причем она как об прибой разбивается об препятствие и например 0.87 волны отражается, а остальное проходит дальше. Пока ты не займешься измерением, эта шняга будет в суперпозиции "прошло не прошло".
505 463222
Как обосновать падение температуры на всей Земле для сюжета комикса? Нужно ясное голубое небо для ламповости, так что летающая сажа от вулканов или ядерной войны подходит плохо.
506 463223
>>63221
Больше интересует сторона процесса исключающая макроскопического наблюдателя.
507 463224
>>63223

> сторона процесса исключающая макроскопического наблюдателя


Суперпозиция двух волн, одна в одну сторону другая в другую
508 463227
>>63218
Корпускулярно-волновой дуализм не предполагает другого объяснения этого же эффекта?
509 463228
>>63222

>Нужно ясное голубое небо для ламповости


Падение активности солнца.
510 463229
>>63224

>Суперпозиция


Это же тоже для наблюдателя, только не взаимодействующего.
511 463230
>>63228
А схуяли, ближайшие миллиарды лет оно только сильнее печь будет
512 463231
>>63229
Это какая-то шутка?
513 463236
>>63231
Нам так препод по физике объяснял, он может немного странный что причёску и усы как у Эйнштейна носит, но говорил вполне серьёзно.
514 463237
>>63236
если вопрос про наблюдателя, то вот https://youtu.be/wZP7mX7dLrI?t=3352
.png102 Кб, 484x516
515 463238
>>63236
Я конечно понимаю, что у волновой функции нет физического смысла. Но если задуматься, то ни одна модель не может ответить на вопрос "как оно на самом деле". А суперпозиция это же реальная вполне вещь и без наблюдателя, если бы её не было, никакой квантовый компьютер бы не смог считать.
Gravitricity - versatile long-life energy storage.mp49 Мб, mp4,
854x480, 2:44
516 463239
Насколько имеется потанцевал?
517 463240
>>63238
Сегодня просто читал:
http://www.sci-news.com/physics/quantum-tunneling-07016.html
Там правда без подробностей, но каким же ещё образом он тогда может проходить через барьер почти моментально.
>>63237
Спасибо, мне есть чем заняться.
518 463241
>>63238
Падажи, как у волновой функции нет физического смысла?
Это вектор состояния, который позволяет связать внутренние/квантовые свойства системы с классической системой наблюдателем.
Собственно мы непосредственно можем мерить только волновую функцию у частиц.
Это все равно сказать, что у фазы радиоволны нет физического смысла.
519 463242
>>63239
ну шахта в 1500м это влажновато
.png68 Кб, 785x463
520 463243
>>63241
Эм... физический смысл. Это про измеряемые величины. Волновую функцию нельзя померить. Квадрат можно, импульс можно, волновую функцию нельзя. Это абстрактный математический объект для расчетов.

> у фазы радиоволны нет физического смысла


Фазу радиоволны мы можем замерить. Волновая функция же принимает комплексные значения.
>>63240

> проходить через барьер почти моментально


Прекрасно. Понять бы еще что это блять значит, в мире, где понятия "траектория" наглухо лишено какого-нибудь смысла. Ты когда электрон в сторону барьера отпустил, до самого измерения не можешь знать где этот хуй (не потому что не знаешь, а потому что этот хуй и правда всюду) и там и тут и в голове твоей и на Марсе. А в самой статье они приводят 4 разных способа определить "время прохождения" которое на самом деле не время, а так, параметр. И в конце концов рисуют даже то, о чём я говорил, начинают складывать по траекториям.
521 463244
>>63242

Это средняя глубина по нефте-газовой промышленности, о чем ты? уже давно на потоке и всё развито.
522 463245
>>63244
Стоит это только дохуя.
523 463248
>>63243
Лол. Об этом говориться в самом начале при формулировке квантовой теории.
По определению значение волновой функции задает коммутирующие величины. Допустим измерили частицу -получили импульс и координату. Некоторое соотношение между ними является некоторая величина эта величина и является значением волновой функции. В некотором роде при идеальном измерении получается, что то что мы измеряем является проекция вектора волновой функции на наш детектор.
То что квадрат модуля волновой функции является вероятностью это уже десятое и важно по сути только для постоянное статистического поведения: в смысле позволяет построить модель идеального газа из квантовых частиц, например из фотонов.

Ах да забыл, у радиоволны есть поляризация, которая связана с фазой. Вместе они образуют комплексный вектор. Вектор полностью соразмерен размерности волновой функции.
524 463249
>>63248
Ты как-то слишком буквально понимаешь фразу "физический смысл". И да, комплексная поляризация тоже абстракция математическая, которую нельзя измерить. Работая в комплексных числах, мы просто не хотим видеть косинусы и синусы, а под конец, мы ВСЕГДА берем либо модуль либо действительную часть, чтобы вернуться в измеряемые величины.
Волновая функция это черный ящик, мы как-то воздействуем на систему, знаем начальные измеряемые величины, знаем конечные величины, проще всего (математически) получить такие предсказания это ввести волновую функцию, но физического смысла она не имеет, просто инструмент.
525 463250
>>63245

>Стоит это только дохуя.



А 10 Мвт лития сколько стоит?
526 463251
>>63250
тоже дохуя
527 463255
>>63207
Ну, может не полностью сожмемся, че нибудь потеряется. Сожмемся из за гравитации, центр вселенной всегда будет горячее и больше окраин
528 463257
>>63209
Какой вопрос, придурок?
529 463258
>>63249
Любой математический объект из физической теории имеет физическое проявление в реальности. К примеру энергия является в своем изначальном смысле интеграл движения, т.е. чисто математический абстрактный объект, но зато какой в него смысл вкладывают, неверно интуитивно интерпретируя, забывая о истинном смысле.

>комплексная поляризация тоже абстракция математическая, которую нельзя измерить


Тебя бы сейчас обхуярил любой радиоинженер. Тебе даже школьник из радиоклуба постоит антенну, которая мерить фазу и поляризацию радиоволны.
На КВ при расчете антенн давно используют вместо фазы и поляризации комплексное число. И этот параметр очень важен, поскольку влияет на диаграмму приема и добротность.

И похоже что-то путаешь. Волновая функция полностью определена на пространстве состояний, каждому состоянию систему соответствует определенный вектор, и зная ее значение в одном состояние мы можем восстановить значение во всех других. Черным ящиком она является только в следующем случае: есть множество AB и C, волновая функция просто задает соответствие этих множеств(функция по определению же). Измерение в квантовой теории это наугад берем из множества C, некоторый элемент. В этом роде черный ящик является множество C, а результат измерений его элемент c. Поскольку функцияотображение тоже является множеством, можно сказать волновая функция образует пространство, а с указывает на вектор функции, который уже указывает на объект из AB. Важно то, что множество AB, C и волновая функция сильно различны. Физика происходит в множествах AB и на пространстве волновой функции, через C только подглядываем. Кстати, граничные условия мы задаем в AB, а не в C. Тут обычно приводят аналогию с разложением в спектр сигнала. AB сигнал. C спектр. преобразование собственно функция.
Ну или совсем для тупых. Звук. Звук это колебания среды, наши уши это детектор. Уши раскладывают звук на соответствующие частоты и мы интерпретируем их как звук. Но ведь есть частоты которые мы не слышим число в силу несовершенства уха(получаем смешанное состояния), это значит они не физические, а лишь математическая абстракция?
529 463258
>>63249
Любой математический объект из физической теории имеет физическое проявление в реальности. К примеру энергия является в своем изначальном смысле интеграл движения, т.е. чисто математический абстрактный объект, но зато какой в него смысл вкладывают, неверно интуитивно интерпретируя, забывая о истинном смысле.

>комплексная поляризация тоже абстракция математическая, которую нельзя измерить


Тебя бы сейчас обхуярил любой радиоинженер. Тебе даже школьник из радиоклуба постоит антенну, которая мерить фазу и поляризацию радиоволны.
На КВ при расчете антенн давно используют вместо фазы и поляризации комплексное число. И этот параметр очень важен, поскольку влияет на диаграмму приема и добротность.

И похоже что-то путаешь. Волновая функция полностью определена на пространстве состояний, каждому состоянию систему соответствует определенный вектор, и зная ее значение в одном состояние мы можем восстановить значение во всех других. Черным ящиком она является только в следующем случае: есть множество AB и C, волновая функция просто задает соответствие этих множеств(функция по определению же). Измерение в квантовой теории это наугад берем из множества C, некоторый элемент. В этом роде черный ящик является множество C, а результат измерений его элемент c. Поскольку функцияотображение тоже является множеством, можно сказать волновая функция образует пространство, а с указывает на вектор функции, который уже указывает на объект из AB. Важно то, что множество AB, C и волновая функция сильно различны. Физика происходит в множествах AB и на пространстве волновой функции, через C только подглядываем. Кстати, граничные условия мы задаем в AB, а не в C. Тут обычно приводят аналогию с разложением в спектр сигнала. AB сигнал. C спектр. преобразование собственно функция.
Ну или совсем для тупых. Звук. Звук это колебания среды, наши уши это детектор. Уши раскладывают звук на соответствующие частоты и мы интерпретируем их как звук. Но ведь есть частоты которые мы не слышим число в силу несовершенства уха(получаем смешанное состояния), это значит они не физические, а лишь математическая абстракция?
530 463259
>>63212У тебя анальная фиксация
531 463260
>>63222>>63239
Нихуя не понятно, но ЕМНИП так глубоко копать землю можно только в избранных местах избранным корпорациям
532 463261
>>63222
Повышения альбедо почти всей поверхности. Например застроили большую часть поверхности хорошо отражающими свет говном.
533 463262
>>63230
Это всё прогнозы. Вполне может произойти какой-то процесс, который изменит их.
534 463264
>>63262
Звезды как и все астрономические тела медленно эволюционируют. Суммарный энергетический масштаб процессов на солнце не находиться в критических состояниях.
Солнце принадлежит в классу стабильных звезд и живут довольно мирной жизнью.
Конечно есть вероятность супервспышки, но при них наоборот солнце начинает жарить сильнее, да еще в рентгене.
Ну или совсем экзотика: пролетает рядом ЧД или нейтронная звезда которую не видели вне плоскости галактики. В итоге орбиты всех планет возмущаются, а некоторые вообще могут быть выброшены из системы. Но этот процесс тоже довольно медленный и займет сотни тысяч лет.
535 463268
>>61534 (OP)
Скажите, вот есть два астероида, между ними будет гравитационное притяжение? Если да, то за счёт чего оно возникнет?
EffetTunnel.gif322 Кб, 200x200
536 463269
>>63243

>Прекрасно. Понять бы еще что это блять значит, в мире, где понятия "траектория" наглухо лишено какого-нибудь смысла.


Ну как, когда электроны подходят к барьеру и начинают интерферировать они интерферируют и с электронами находящимися на границе барьера которые смещаются, а часть подходящих встаёт на их места и те по цепочке смещают соседние, соответственно с обратной стороны получается почти моментально выталкиваются крайние, может ведь так быть. Это не то чтобы как бывает что кому-то лишь бы поспорить, просто физикой больше как увлечением интересуюсь.
537 463270
>>63269
А если один электрон?
538 463271
>>63270
Интерференция возникает из-за волновых свойств, а волновые свойства возникают при определенных условиях, при которых количество частиц роли не играет. Даже при одной электроне будет.
539 463273
>>63258
Ты считаешь всех собеседников тупыми? К чему мне эти очевидные вещи? Ты просто пересказываешь учебник. Про Гильбертово пространство состояний и т.д. и т.п.

> Но ведь есть частоты которые мы не слышим число в силу несовершенства уха(получаем смешанное состояния), это значит они не физические, а лишь математическая абстракция?


Именно так. Ведь измеряем мы амплитуду, а не частоту. Частота это уже наша обработка. А вместе с фазой, хоть комплексную амплитуду можно придумать. Ты пытаешься вложить в категорию "физический смысл" бытовое понятие. Даже создается впечатление будто ты веришь что волновая функция существует и "так оно там на самом деле и происходит".
540 463275
У волновой функции нет физического смысла.
Это как придумывать физический смысл действию. Не нужно вскрывать эту тему, как говорится, и начинать искать аналоги из реального мира для некоторых конструкций, так можно очень сильно объебаться с понималкой.
541 463277
>>63273

>К чему мне эти очевидные вещи?


К тому, что 90% обитателей сайентача их не знают.
мимопроходил.
542 463278
>>63269

> может ведь так быть.


Ты рассуждаешь об электронах как о частицах, которые "реально" двигаются куда-то. Это, конечно, не так.
543 463279
>>63273

>Даже создается впечатление будто ты веришь что волновая функция существует и "так оно там на самом деле и происходит"


Лол, а ты можешь однозначно утверждать, что это не так? Есть мнение, что ты под слово "физический смысл" пытаешься какие-то свои аналогии приплести, ученые так раньше пытались электродинамику на механических аналогиях построить, нихуя не вышло. Возможно, что такая математика и описывает максимально точно этот самый "физический смысл", просто она работает на слишком низкоуровневой логике, чтобы человек мог это осознать.
544 463280
>>63273

>Даже создается впечатление будто ты веришь что волновая функция существует и "так оно там на самом деле и происходит"


Лол, а ты можешь однозначно утверждать, что это не так? Есть мнение, что ты под слово "физический смысл" пытаешься какие-то свои аналогии приплести, ученые так раньше пытались электродинамику на механических аналогиях построить, нихуя не вышло. Возможно, что такая математика и описывает максимально точно этот самый "физический смысл", просто она работает на слишком низкоуровневой логике, чтобы человек мог это осознать.
мимо
545 463281
>>63271
Нет, погоди, энергия от столкновения одного электрона с барьером, должен бы быть поглощён барьером, причём тут интерференция?
546 463282
>>63275
А как же эти ваши кубиты? Они тоже не имеют физического смысла, раз они напрямую связанны с волновой функцией?
547 463285
>>63279
А может и бог есть, а может и нет. Может описывает, а может и нет. Какая то ебанутая аппеляция к недоказуемости.
Ты сначала покажи этот физический смысл и нечего писать поток своих мыслей. Физического смысла никакого нет у волновой функции. Волновая функция появляется в строго определенном месте и она является плодом работа математиков и физиков, чисто наше изобретение.
548 463286
>>63285
Физический смысол - это в любом случае перекладывание физических законов на понятные человеку понятия.
549 463288
>>63282
ну как бы его можно описать в том числе с помощью волновой функции, в чем проблема-то.а так физический смысол кубита это двухуровневая система, любая, хоть из говна и палок, только б работало.
550 463289
>>63212

>внимание


Софистика у тебя получается какая-то. Вводишь термин и становишься его заложником.
551 463290
>>63286
Нет. Все описание строится из понятий и терминологий и так, так что аргумент не работает. Если смотреть ретроспективно, то из наблюдения физических явлений пытались выводить некие законы и приписывать им некие параметры. С расширением мат. аппарата стали появляться чисто утилитарные вещи, такие как волновая функция, действие, вектор и тд., которые не находят аналогов в физическом мире. Так же с прогрессивным развитием науки стало возможно переобозначать или описывать совершенно новые физические явления. И если вот есть такая биекция между чисто феноменологическим и теоретическим, то это самое обладает физическим смыслом.
552 463291
>>63281

> должен бы быть поглощён барьером


Там энергии намноооооого меньше чем релятивизм, при котором вообще появляется вероятность возникновения новых частиц.
553 463293
>>63291
Ниче не понял
554 463295
>>63278
Нет, они конечно могут двигаться и "нереально" и не как частицы, но двигаются же, или как тогда?
556 463299
>>63261
Чё, реально снег и льды на всей планете после атомной зимы отражением может удерживать температуру ниже нуля тысячи лет?
Форма галактик 557 463300
Темная материя придает форму галактикам? Их вроде три вида спиральные, эллиптические и неправильные. Ведь в скопления темной материи упала барионная материя и образовала галактики?
558 463301
>>63296
Да-да, натуральная анальная фиксация
559 463308
>>63222
Затухание активности ядра Земли. Как следствие, уменьшение температуры на планете, ослабление магнитного поля, пизда большей части живности.
560 463309
>>63299
Атомная зима довольно сомнительная вещь, ядерное оружие переоценено.
На Земле уже было глобальное оледенение, ледник доползал почти до экватора. Кстати, мы живем в эпоху локальной оттепели или межледниковья. А каких-то 50-60 лямов лет назад в Антарктиде были субтропики.
561 463314
>>63309
Бля ледниковый период слабо подходит, все просто свалят в южную часть полушария с нормальной температурой
>>63308
Ядро остынет-магнитное поле ослабнет-атмосферу частично сдует солнечным ветром-радиация солнца убьёт всё нахуй, это уже перебор
562 463315
>>63299
А ты не думал о том, как в таком случае будет синтезироваться кислород? Все леса сдохли от постоянного холода, океаны скованы льдом. Чем дышать?
563 463316
>>63314
В южном полушарии меньше суши, и тепло только у экватора и побережья.
564 463319
>>63314

>свалят в южную часть



Там тесновато
565 463322
Буду очень благодарен, если мне смогут привестм и объяснить эксперименты по определению спина фотона. У меня уже крыша едет сколько всего написано, но как он вот фактически определяется не могу найти. Только про электрон нашёл, что две полоски на мишени образуют после прохождения через магнитное поле и это доказывает наличие у него спина, про фотон ничего не нахожу. Помогите
566 463324
>>63322
Странно что ты ничего не нашел, это чуть не первая тема которая в квантмехе обсасывается при рассмотрение спина.
Спин у фотона это поляризации волны, определяется так же как у электрона через спиноры.
Доказывается спин у фотона это тупо поляризаторами и лазером. Есть более сложный опыт по разделению луча на два луча с разными спинами. Такие лучи кстати запутанные.
567 463327
Почему электрическое поле квантуется, а магнитное - нет?
568 463330
>>63327
Электромагнитное квантуется
569 463336
>>63314

>это уже перебор


Ну а ты возьми начальную стадию, когда ядро только начало остывать, температура падать, но глобальный пиздец еще не пришел.
570 463345
Возможно ли хотя бы теоретически принять сигнал электромагнитной волны на частоте теплового шума, который по амплитуде меньше его максимума?
Антенны, охлажденные до околонуля, допускаются.
571 463346
Парни, парни, слушайте чё придумал.
Как холодильник, но наоборот.
В квартире радиатор, который греется, а на улице тот, который охлаждается.
Вот допустим на улице -20, а газ в радиаторе на улице при расширении охлаждается до -70, например. Греется до -40, например, более тёплой атмосферой. Поступает обратно в квартиру, сжимается компрессором и отдаёт забранное с улицы тепло.
Этож получается можно получить больше энергии, чем если просто греться электричеством из розетки.
Пусть лучше это электричество гоняет компрессор, который ворует тепло с улицы.
Наверняка есть газ, который подойдёт для такого нагревателя.

Взлетит? И кто-то уже пытался такое сделать?
572 463350
>>63346
https://ru.wikipedia.org/wiki/Тепловой_насос
Современные кондиционеры уже давно как это умеют, покупай и пользуйся на здоровье.
573 463351
>>63346
Кстати тут вполне себе возможен перпетуум мобиле - система-то не замкнута, кидаешь радиатор под землю, второй на воздух, от разности температур берешь энергию и направляешь ее в свой дом повышая температуру выше чем в грунте и воздухе.
574 463353
>>63351

>Как выяснили ученые, температура поднимается на 3 градуса каждые 100 метров вглубь Земли. Эта цифра является постоянной для всех континентов и частей земного шара.


Осталось придумать как закопать огромный радиатор на глубину в пару сотен метров.
575 463355
>>63353
Элементарно: находишь ближайшую заброшенную шахту, и заливаешь водой(если она еще не затоплена). А далее откачиваешь воду и пропускаешь сквозь свой радиатор.
Это самое простое и первое что пришло в голову. Вариантов море, было бы желание.
576 463356
>>63355

>заливаешь


>откачиваешь


>пропускаешь


Столько энергии просрать, чтобы получить тепла в несколько градусов. Охуенный вечный двигатель, сразу нобелевка.
577 463358
>>63330
Ну окей, так почему магнитное не квантуется
578 463359
>>63356
Да там вон чё придумали.
Закапывают шланги, по ним пускают солёную жижу, которую загоняют в тепловой насос, который и есть холодильник с фреоном.
579 463361
>>63358
Ну так у тебя квант магнитного поля, как частного случая электромагнитного, - фотон, не?
580 463363
581 463364
>>63358
Гугли: квантование магнитного потока.
Такие задачи студентам обычно дают.
582 463375
>>63351
Поздравляю, ты только что ГеоТЭС.
583 463383
>>63359
Так а в чём суть? Конвертируют постоянные +4 под землёй с большой площади в +22 на меньшей площади в доме?
584 463384
>>63330

>Электромагнитное квантуется


Квантование же это просто допускаемое упрощение для расчётов.
586 463386
>>63384
Не совсем, есть ряд косвенных подтверждений реального существования виртуальных частиц как бы тупо данная фраза ни звучала
Например, эффект Казимира и излучение Хокинга.
sunspot21.jpg19 Кб, 400x300
587 463387
>>63230
Нет, даже в процессе прибывания на главной последовательности, т.е. во время своего относительно стабильного состояния, у звёзд может довольно значительно изменяться активность.

И это колебания за совсем короткий период времени, если посмотреть в масштабе тысяч, десятков и сотен тысяч лет, то хер знает насколько сильно его там колбасило.
588 463388
>>63257

>Какой вопрос, придурок?


Не придурок, придурок ты если не уловил суть такого простого объяснения.
589 463389
>>63384
Если бы квантование было бы допускаемое упрощение. То мы бы не смогли наблюдать порционность полей.
>>63386

>эффект Казимира и излучение Хокинга


Эээ... эффект Казимира это ван-дер-ваальсовы силы, просто рассчитанные иначе.
590 463390
>>63184
Но ты ведь просто прибавляешь к кучке нулей ещё нули, сколько нулей не складывай сумма всё равно останется ноль.

> Энергия поля растёт по фатку его расширения.


Эта энергия равна нулю, в любой точке поля, в любом объеме и в любой момент времени.
591 463391
>>63389

> Эээ... эффект Казимира это ван-дер-ваальсовы силы, просто рассчитанные иначе.


Охуительные, блять, истории, а может расскажаешь, почему эти ван-дер-ваальсовы силы работают только между зеркальными и идеально параллельными поверхностями, а стоит их повернуть хоть на угловую секунду, так силы сразу исчезают?
592 463392
>>63386

>есть ряд косвенных подтверждений


>излучение Хокинга


Косвенно подверждаем косвенными догадками о косвенных процессах возможно косвенно происходящих.

Кстати, хорошо что напомнил.

Ну вот парное появление одной частицы и одной античастицы возле горизонта ЧД. Ну допустим одна, античастица, за горизонтом появилась, другая, частица, до горизонта и улетела.
Но чисто статистически же в половине случаев будет наоборот, частица появится за горизонтом, а античастица до.

Схуяли вообще будет потеря массы чд?
593 463393
>>63389

>То мы бы не смогли наблюдать порционность полей.


А как мы его наблюдаем?
594 463394
>>63392
Я надеюсь ты в курсе, что у античастиц также положительная масса? То есть да, чёрная дыра равномерно излучает как частицы, так и античастицы, и те и другие забирают у неё массу.
595 463395
>>63390
Хмм, да, похоже я не так просто об этом в тот момент думал.
Так как я писал было бы в случае процесса рождения частицы и исчезновения частицы.
Но происходит-то рождение частицы и античастицы.
Т.е. берётся ноль, из него делается -1 и 1, а потом опять ноль.
Нулей можно взять бесконечно ничего не изменяя в целом.
А в моём изначальном рассуждении брался 1 и превращался в 0. Тогда бы чтобы взять одновременно больше единиц, должно было расти общее чего-то там.
596 463396
>>63394
Так половина же стабильно оказывается внутри неё и только половина улетает.
598 463398
>>63396
Именно так, те что падают внутрь получают как бы отрицательную массу, которая берётся взаймы для тех, что улетают наружу и получают положительную массу, то есть становятся реальными. Более подробно это не объяснить без помощи кучи сложных формул.
599 463400
>>63397
Почитал, весьма любопытно, жаль, что там подробно не обсуждается тема собственных колебаний поля, которые упомянуты как раз в частном случае эффекта Казимира. Но если я правильно понял, их выкладки позволяют объяснить его без помощи виртуальных частиц, что исключает этот эффект из косвенных доказательств.
600 463402
>>63398

>Более подробно это не объяснить без помощи кучи сложных формул.


Если это происходит, значит можно.
Происходит-то оно не с помощью сложных формул.
601 463403
>>63402
Хохо, такого сильного заявления я ещё не слышал, опровергать его я конечно не буду.
602 463404
>>63400

>их выкладки позволяют объяснить его без помощи виртуальных частиц


Так виртуальные частицы возникают как раз в физике при финтах с математикой. Я вот к чему веду. (И они вроде там пользуются ими, они же диаграммы рисуют)
603 463405
>>63403
Но всё так и есть. Если не можешь описать что-то словами, то плохо знаешь тему, а просто бездумно заучил формулки и выводы.
Формулы ведь просто расписывают суть.
604 463406
>>63404
Я это знаю, но сейчас использование виртуальных частиц в расчетах зашло настолько далеко, что некоторые процессы не удаётся объяснить без их использования, может в принципе это и возможно, а может и нет, вот в случае с Казимиром оказалось возможно, плюс они вылазят чуть ли не напрямую, как с излучением Хокинга, реальные частицы рождаются просто изнихуя, почему рождаются именно такие частицы, именно в таком соотношении и количестве? Не уверен, что на эти вопросы можно ответить, при помощи модели исчисления, не использующей виртуальные частицы.
605 463407
>>63405
Нет, я конечно могу заменить переменные в формулах на слова и сказать например, что мощность излучения Хокинга равна приведённой постоянной планка, умноженной на скорость света в вакууме в шестой степени и поделённая на произведение пятнадцати тысяч трёхсот шестидесяти, числа пи, квадрата гравитационной постоянной и квадрата массы чёрной дыры, но боюсь, что так будет ещё непонятнее.

Не на любой вопрос можно ответить без помощи формул, например, если ты спросишь, чему именно равна мощность излучения Хокинга, то на этот вопрос совершенно точно железно никак невозможно будет ответить без представления формулы в том или ином виде.
606 463408
>>63407
На вопрос о конкретной величине-конечно нет, а на принципиальный вопрос почему оно вообще так-да.
607 463410
>>63408
Есть мнение, что многие процессы человек уже не может понять без формул и мат.моделей, настолько это отличается от привычной реальности, а любые попытки разъяснить их - лишь аналогии.
608 463411
>>63406
https://youtu.be/snwdHjZODf0
Тут есть про реальность виртуальных частиц.
609 463412
>>63411
И да, в саентаче наконец-то кидают людям лекции, а не научпоп-говно. Можете начать аплодировать.
610 463422
>>63356

>заливаешь


Тут энергию получаешь.

>откачиваешь


Тут ее же тратишь

>пропускаешь


А тут не тратишь практически нихуя.

>Столько энергии просрать, чтобы получить тепла в несколько градусов.


Тепло ты в джоулях получаешь, и можешь получить довольно много, хватит не только на насосы.
В любом случае какая-то циркуляция жидкости/фреона в системе будет.

>>63383
Да. По сути та же землянка только 2.0.
611 463423
>>63411
Посмотрел, тут как раз эффект Казимира приводится в качестве доказательства и относительно подробно разбирается, при этом никаких Ван-дер-Ваальсовых сил не упоминалось, хотя лекция вроде свежая. То ли та статья не является признанной научным сообществом, то ли лектор про неё не в курсе, то ли ещё что, хз.
612 463425
>>63345
Бамп вопросу
613 463426
>>63345
Под тепловым шумом подразумевается реликтовое излучение?
614 463429
>>63423
Лектор приводит в качестве аргумента существования виртуальных частиц, эффекты, которые объясняются виртуальными частицами. Но вообще в чём суть, вот у нас есть какая-то система, мы хотим посчитать силу. Записываем уравнение Шредингера, которое в силу причин просто невероятно сложно решить. Но теоретически у него есть решение, и никаких виртуальных частиц в этом решении как бы нет. Но так как решать такую хуиту никто не может, то начинают пользоваться теорией возмущений, где каждый член можно представить как какое-то конкретное взаимодействие реальных частиц и виртуальных. Вот тут и возникает вся суть.
615 463430
>>63315
бля пиздец. Ну и чем дышать?
616 463432
>>63429
Это всё понятно, но вдруг такие приближения не совсем верным и при честном решении уравнения Шредингера получатся немного другие результаты? Или наоборот, оно окажется более верным, а при честном решении никаких излучений Хокинга и эффектов Казимира не возникает, тогда это будет сильным доказательством.
617 463433
>>63430
Ребризеры в помощь.
618 463435
>>63432

>при честном решении никаких излучений Хокинга и эффектов Казимира не возникает, тогда это будет сильным доказательством.


Не... честное решение как раз должно давать теорию с огромной точностью.
f4e.png695 Кб, 666x666
619 463436
>>63433
Брутальненько
620 463440
>>63422

>Тут энергию получаешь.


>А тут не тратишь практически нихуя.


Какая-то маняфизика, ей богу. У тебя на один только насос, гоняющий все это по кругу, будет уходить энергии больше, чем получится выхлопа.
621 463462
>>63426
Скорее тепловое излучение в атмосфере при комнатной температуре.
Впрочем, это не столь важно, пусть будет и реликтовое, или вообще любой шум.
Вопрос в том можно ли принять сигнал с амплитудой ниже уровня шума, путем нахождения каких-то закономерностей например.
622 463464
Верно ли, что гены по сути лишь набор инструкций о том, как и когда начинать-прекращать делать белки ? И никакой явной информации о строении будущего организма в них нет (её можно узнать только в ходе опытов и симуляций роста)?
623 463467
>>63462
Если шум не рандомный, то можно, но тепловой шум обычно всегда рандомный, так что нельзя. Вернее принять-то можно сам факт сигнала, ибо среднее вырастет, а вот расшифровать - нет. Хотя если сигнал будет повторяться по многу раз, то можно будет вычленить. Но это уже из области криптографии, кмк.
>>63464
Истинно так.
624 463470
>>63464
Гены это только чертежи белков. Собственно ген по определению это кусок ДНК в котором закодированы последовательности аминокислот, либо РНК на рибосомы/прочую лабуду.
А когда начинать-прекращать делать белки(это называется экспрессией генов) уже определяется огромным множеством факторов, в первую очередь реакцией белков на внешние условия среды, или наличие съемных заглушек на ДНК(факторы транскрипции).
Собственно сборка белков делается из матричной РНК, а ее делает белок(РНК полимераза), который "бегает" вдоль ДНК. Но на ДНК обычно навешиваются всяческие молекулы (от простых молекул до сложных белков и РНК цепочек), которые либо "ставят на паузу", либо заставляют соскочить с ДНК или наоборот наводить РНК-полимиразу на нужное место в ДНК.
В некотором роде предсказать строение организма можно, главное знать как каждый кодируемый белок в зависимости от условий среды влияет на экспрессию генов.
625 463472
>>63467

>Если шум не рандомный, то можно, но тепловой шум обычно всегда рандомный, так что нельзя.


Шум рандомный, а вот сигнал - нет. Задача сводится к нахождению в шуме нужных сигналов.
Понятно что 1 полезный сигнал + рандомный сигнал на этой же частоте = рандомный сигнал. Однако несколько таких сигналов на разных частотах уже не будут рандомом, ведь вероятность того что определенная комбинация сигналов одновременно появилась весьма мала - а значит это наш сигнал. Это как пример моих размышлений.

>Но это уже из области криптографии


Да, задачка больше про информатику чем про физику радиоволн. Однако и про физику тоже - возможно ли вообще принять очень слабый сигнал сигнал, который ниже чем фоновый шум?
И если да, то какова тогда вообще теоретическая нижняя граница приема радиоволн и чем она обоснована?
626 463474
>>63472

>Да, задачка больше про информатику чем про физику радиоволн


Ну, не совсем. Тут очень много всякой мути с шириной спектра, временем передачи, временем приёма, всякими соотношениями. Теория обработки сигналов как раз про всё это вот рассказывает.
мимо
627 463475
>>63474
Вопрос и про физику и про информатику, а так же про энтропию(двух типов), демона Максвелла, потерю информации и вот это вот все.
Понятно что налаживание стабильной связи подобного типа - задача не одного поколения НИИ, вопрос про теоретические пределы подобного. Ну или может уже кто-то подобное уже изобрел, а я просто не в курсе как вон тот анон не в курсе про существование обратных кондиционеров.
628 463483
>>63475
Давно есть такие методы, например используются для связи с далекими спутниками и марсоходами. На них большую антенну не поставиш и мощный приемник не засунеш. Но за всё приходиться платить, в данном случае за счет медленной передачи данных. Т.к. одну и ту же информацию передают многократно и с большой избыточностью.
629 463485
>>63483
Это понятно, но там прямая видимость(марсоход не шлет напрямую на Землю, а сначала на спутник Марса), да и шума не так уж много, космос все-таки, холодный, безжизненный и вакуумный.
Совсем другое дело - когда у тебя сигнал ниже уровня шума, при чем бума в идеале белого.
Интересует нижняя граница возможности приема маломощных сигналов.
В теории, кстати, фотоны летят бесконечно далеко, и практически сквозь все на радиочастотах. А значит поидее можно принять, скажем, wifi твоего роутера где-то в Южной Африке или США. Чем ограничена такая возможность?
630 463486
>>63483

>за всё приходиться платить, в данном случае за счет медленной передачи данных.


Получается чем выше частота, тем больше энергии рассеивается, тем меньше энергии доходит, тем слабее сигнал, тем меньше скорость передачи информации.
Итого имеем некий максимум отношения частоты несущей к максимально возможной скорости передачи информации.
631 463487
>>63485
Волны вообще-то в средах затухают: фотоны атомами поглощаются и возбуждают их.
632 463489
>>63485

>Чем ограничена такая возможность?


1. Стандартный вайфай ты не примеш, нужны специальные протоколы и сигналы, которые в домашних вайфая не используются
2. Предположим организованы эти самые сигналы, то ограничение снизу очевидна - чувствительность приемника. Но при приближении принимаемого уровня сигнала к уровню чувствительности скорость и объем передачи стремятся к 0.
633 463495
>>63487

>фотоны атомами поглощаются и возбуждают их.


Тоесть, нижняя граница приема - это когда все фотоны с большой вероятностью будут поглощены, а переизлученные так же с большой вероятностью не найдут путь к приемнику?

>>63489
1.Я про несущую хотя бы.
1.1.Почему же они не используются если бы это помогло увеличить дальность в обмен на скорость?
2.Хорошо, а чем ограничена чувствительность приемника? Можно ли принять 1 фотон(как неделимый квант э/м волны), или эта граница выше?
2.1. А если у нас теоретически идеальный приемник, каковы еще ограничения?
634 463498
>>63388
Так вопроса или объяснения, мудила ты ебнутый?
635 463499
>>63495
1.1 Потому что никому не нужна такая дальность вайфая.
2. Специальным приёмником можно, обычной антенной нет, у специальных приёмников есть ряд ограничений.
636 463504
>>63495

>Почему же они не используются если бы это помогло увеличить дальность в обмен на скорость?


Потому что не хотят получить вайфай со скоростью модема.
637 463574
>>61534 (OP)
Объясните, пожалуйста, почему вообще работает третий закон Ньютона. Да, я понимаю, что это эмпирический закон, но ведь наверняка за столько лет с момента его открытия где-нибудь появилось какое-то обоснование, связанное с квантовой физикой/химией или каким-то иным непонятным колдовством.
638 463586
>>63574
Сила, это производная энергии по координате. Теперь смотри какая тема, если у тебя есть два кусочка, они образуют систему. Они как-то взаимодействуют. Вопрос в том, какая производная у энергии, при изменении расстояния между этими двумя кусочками? И тут возникает такая ситуация, что формула расстояния между двумя точками симметрична к перестановкам.
639 463590
>>63574
Лол, это самое начало теормеха, где выводятся все законы ньютона из принципа наименьшего действия.
Вот модель классической механики для гуманитариев: вся физическая материя представлена из частиц. Частицы сами непосредственно не могут взаимодействовать, для этого нужно другая сущность поле. Сущность поля это некоторая величина определенная по всему пространству времени. Частица в процессе своего движения постоянно "корректирует" свой курс, сверяясь с величиной поля, при сверке частица сами оставляет "отпечаток" своего движения на величине поля, тем самым давая другим давая другим частицам узнать о себе. Отпечаток имеет определенное соответствие с величиной корректировки движения, и это соответствие симметрично.
Еще раз, частица двигается, смотрит на поле, и в зависимости от своего движения корректирует своего движение, при этом не забыв "деформировать" поле. Эта хуитень называется второй закон Ньютона.
Поле злое и помнить все, как надругались над ним. Поле ничего не прощает и не забывает и сполна отдает другим частицам в назидание. Если процесс непрерывный и симметричный (а это зависит от пространства), то частица действуя на другую частицу встречает равное противодействие, получаем наш третий закон Ньютона.
640 463643
>>63222
Бойзы, я тут подумал ещё, что насчёт пролетающей мимо блуждающей планеты на огромной скорости, что своей гравитацией смещает орбиту Земли, заставляя её сильно отдаляться от Солнца с каждым оборотом. Это реально вообще?
641 463644
>>63643
Нет, от такого резкого смещения вся поверхность планеты пойдёт по пизде, ну разве что это будет идти постепенным процессом несколько тысяч лет, но всё равно при каждом пролёте будут лютые цунами.
642 463658
Человек может на постоянной основе жить под землёй? Какие подводные?
643 463664
Верно, что мозг не очень приспособлен для познаний и личностного развития? Т.е. если б природе было нужно, могла бы сделать намного лучше? А так приходится упорно избавляться от когнитивных искажений, примитивных социальных установок и тд. Или эти проблемы свойственны всем познающим субъектам принципиально?
644 463667
>>63664
надо спросить у дяди Анохина
Петька Анохин 645 463668
>>63667
Я здесь, родной, чего хотел?
646 463680
>>63658
Ты хотел сказать подземные?
Если организовать АЭС, то в принципе может, не вижу препятствий.
>>63664
Верно, есть теория, что из-за быстрой эволюции коры больших полушарий у них хуёво проработана связь с эволюцонно более древними отделами мозга. Из-за этого возникают противоречия, самое главное из которых - лень, современное общество настолько привыкло к тому, что человек постоянно должен заставлять себя что-то делать, что не замечает, что это вообще-то ненормально и неестественно. Если бы человек поэволюционировал ещё пару миллионов лет, то эти связи бы наладились и таких проблем бы не было, логичного обоснования необходимости какого-либо действия было бы достаточно для побуждения к его исполнению.
647 463681
>>63590
Какой бред, бедные студенты
648 463682
>>63643
Да, но там не только скорость, но и масса ее и расстояние важны. И ее траектория тоже изменится
649 463683
>>63658

Сам по себе нет, или он эволюционирует до нужной кондиции и перестанет быть человеком.

А если полностью сымитировать условия на поверхности земли, то да
650 463690
>>63681
А есть еще Гамильтонова механика, там еще хуже. Вместо частиц вводиться пространство состояний, а видимые движения всего лишь "проекция" пространства состояний на систему отсчета.
651 463696
>>63658
Чё жрать то под землёй?
03f9cf351a727686a1d95cd2475795c1--memes-pepe-the-frog[1].jpg8 Кб, 236x231
652 463697
>>63644
>>63682
Бля почему так сложно сделать твёрдый Frostpunk?
653 463700
>>63696
Теплицы с искусственным освещением.
654 463737
Верно ли, что базовые эмоционально-ассоциативные установки из раннего детства формируют устойчивые нейронные структуры, которые повторяют себя во всём дальнейшем развитии и являются "единицами информации" в мозге? Например, любили или ненавидели, чувствовал ли безопасность и тд.
655 463739
Или, может, это лишь для психотравм свойственно? Типа доминирующей структурой устанавливается авторитарный режим без обратной связи, который стремится всё лепить по своему образу и подобию. ну короче хуйню наговорил, ну да в уже тонущем можно
>>63737
656 463743
Если у электрона есть масса, то почему он не падает на ядро?
657 463744
>>63743
Не стоит вскрывать эту тему, ты молодой, шутливый, тебе всё легко...
658 463746
>>63743
А почему должен?
659 463747
>>63743
Если у Луны есть масса, то почему она не падает на Землю?
660 463748
>>63747
Ну у неё же ускорение только в центр масс, значит, она теряет энергию по чуть-чуть. И когда-нибудь ебнулвсь бы, если бы была достаточно близко
661 463749
>>63743
Д И С К Р Е Т Н О С Т Ь
И
С
К
Р
Е
Т
Н
О
С
Т
Ь
энергии
662 463750
>>63748
Неа, Луна наоборот отдаляется из-за приливных сил.
663 463756
>>63668
расскажи еще про мозг
664 463769
>>63743
Потому что электрон это волна.
665 463772
>>63743
Потому что он и так УПАВШNM
666 463845
>>63680

>Из-за этого возникают противоречия, самое главное из которых - лень


Вот тоже хороший вопрос: лень - это потребность, инстинкт, называйте как хотите. И другие инстинкты можно отключить временно или постоянно(например, кастрация лишает тебя сексуального влечения). Можно ли как-то отключить лень? Ну или притупить ее хотя бы(полное отключение вряд ли приведет к чему-то хорошему).
Алсо, прокрастинация(недостаток внимания) - результат лени или это отдельная хуйня?
667 463857
>>63845
Наверняка можно, у тех же аутистов она сильно притуплена и у азиатов
Может даже есть какие-то препараты, за это не в курсе.
668 463861
>>63750
Чтооооо?
669 463866
>>63861
Вот такие дела, можешь просветиться: https://ru.m.wikipedia.org/wiki/Приливное_ускорение
670 463872
Если сшить вместе двух близнецов, начнётся иммунная война или норм пройдёт?
671 463878
>>63872
Если однояйцевых, то не должна.
672 464011
>>63845

>Можно ли как-то отключить лень?


Не заниматься бессмысленным говном которое тебе не интересно, причём зачастую направленное на обогощение других.
673 464012
>>63845

>прокрастинация(недостаток внимания)


Это ж разные вещи.
674 464020
Вот смотрите. Допустим, мы будем двигать что-то очень тяжёлое, настолько, что при всех наших усилиях оно так и не сдвинется с места. Получается, что работы не происходит, энергия ни на что не тратится. Но мы ведь устанем, если будем так пытаться, не?
675 464021
>>64020
Тратится на поддержание мышц в "не нормальном" состоянии.
Нормальное для них расслабленное.
Но тут нужно глубже двигаться.
676 464033
>>64021
Ну эти затраты в мышцах по идее нужны, чтобы организм какую-то работу совершал. Но что именно совершается, если ничего не двигается (на макроуровне, по крайней мере)? Даже если мышцы напряглись, то потом они тоже не двигаются. Мб энергия просто в тепло уходит как-то тогда?
677 464081
>>64033

>Мб энергия просто в тепло уходит как-то тогда?


Так и происходит, мышцы в напряжённом состоянии производят тепло.
678 464085
>>64081
Мне просто кажется, что не настолько много, чтобы достаточно быстро уставать. Вот попробуем мы сдвинуть танк с места: устанем через секунд 10.
679 464086
>>64085
Практически не совершив при этом мех.работы, не считая первоначального напряжения мышц.
680 464093
>>64085
Ну у тебя выбора нет, у тебя есть второй закон термодинамики.
681 464139
>>64093
Ну, может там не всё так просто, может есть какие-то механизмы, которые в этот момент дофига энергии жрут, про которые я не знаю. Ясно дело, что всё либо в мех.работу идёт, либо в тепло.
682 464148
>>64139
Эт надо к биологам или типа того, тут наверное больше физики.

Вообще что такое усталость-это накопление в мышцах какой-то кислоты, молочной вроде.
Осталось понять откуда она берётся, может с током крови, может когда мышцы напряжены через них больше крови качает. Но вряд ли, скорее всего она прямо в них вырабатывается именно при напряжении, а увеличенный ток крови через них при этом чтобы больше кислорода доставлять.
Говорю же, нужно глубже двигаться, что такое мышца, из каких клеток состоит, как клетка сжимается, что при этом происходит, какие процессы и что для этого нужно и т.д.

Электромагнит кстати тоже когда включен постоянно энергию жрёт, хоть работы и не совершает, а только держит что-то.
683 464157
>>64148
Ну он на джоулевы потери ведь тратит, т.е. на работу поля.
684 464182
>>61534 (OP)
http://journalofastrobiology.com/Mars5.html
https://2ch.hk/news/res/4897085.html (М)

Господа вот это вот щитова за то, что жизнь на Марсе таки есть? Или надо ещё посылать туда анализатор что бы точно подтвердить?
саентач1.png16 Кб, 855x432
685 464501
кушал гречечку с лучком и грибочками и ВДРУГ накрыл вопрос пикрелейтад
686 464503
>>64501
Если температура равная изначально. То нет.
687 464708
>>64503
у зеркальной поверхности - типа серебряной плёнки - одинаковый коэф.отражения ИК-излучения с обеих сторон?
Ну возьмем например 100-атомный слой серебра в кач-ве зеркала.
С одной стороны серебро подкислилось и возникла оксидная неотражающая плёнка. Покрасим её в черный цвет.
Фотоны падают на черную поверхность, по слою начинают бежать фононы. Они "выходят" с зеркальной стороны слоя и излучают внутрь камеры ИК фотоны.
Есть поток фотонов снаружи-внутрь - ИК, падающий на черную обратную поверхность зеркала.
И есть обратный поток - те, что не отразились от зеркала изнутри и превратились в фононы.
Так вот я не врубаюсь, почему не возникает градиента - и почему в центральной камере не будет выше температура газа - если как бы "внутрь" идёт больший поток фотонов (греющих газ в центральной камере), чем вылетает обратно. А раз наружу вылетает меньше фотонов, то по идее они вообще почти все должны скопиться в центральной камере.
Это очевидно противоречит 2му началу. А энтропии я верю больше своих глюков.
Не могу только понять, где ошибка - почему баланс фотонов уравновешивается?
688 464709
>>64708
Проверь свои рассуждения еще раз. Но в обратном порядке запусти все процессы.
689 464724
>>64709
не понимаю, "изотропный" же процесс.
укажи плиз на ошибку молю отсосу писос.
Дельта фотонов есть?
690 464741
>>64708

> Фотоны падают на черную поверхность, по слою начинают бежать фононы. Они "выходят" с зеркальной стороны слоя и излучают внутрь камеры ИК фотоны.


Если происходит этот процесс. Значит происходит и обратный процесс с такой же интенсивностью.
И да, зеркало плохо излучает ИК, идеальное зеркало вообще не излучает тепловое излучение например. Посмотри на каждый кусок своего рассуждения, во всех местах у тебя потоки энергии одинаковые в обе стороны должны быть.

>Они "выходят" с зеркальной стороны слоя и излучают внутрь камеры ИК фотоны.


Тут такая же вероятность события, как и вероятность поглощения зеркалом фотона. Так как если ты запустишь время назад в процессе "фотон поглотился зеркалом и упал на его черную поверхность." то получишь процесс "фотон излучился обратной поверхностью и вылетел в камеру".
691 464747
У меня зрение -5. Подошёл к зеркалу в фитнес клубе, смотрю в отражении на EXTRA THICC жопу шмары на тренажере вдали, почему в отражении всё так же хуёво видно? Зеркало же в 5 см от меня, а вблизи я вижу хорошо. Сложнаааааа
692 464748
>>64747
Потому что изображение вдали.
693 464750
>>64748
изображение в 5 сантиметрах
694 464751
>>64750
Изображение - это точка, откуда лучи света исходят. Ты на него глаза фокусируешь.
23819230[1].jpg132 Кб, 630x630
695 464752
>>64751
исходят из отражения в 5см
696 464759
>>64752
Нарисуй картинку, как лучи от объекта до твоих глаз доходят.
697 464900
>>64741

>>Если происходит этот процесс. Значит происходит и обратный процесс с такой же интенсивностью.


В этом видимо моя ошибка.
Не знал, что:

>>зеркало плохо излучает ИК, идеальное зеркало вообще не излучает тепловое излучение например.


Не совсем понимаю, почему так, а электродинамику сплошных сред не успел пройти в универе.
На пальцах можешь примерно почувствовать пояснить - почему не излучает?
698 464961
>>64900
Я электродинамику сплошных сред тоже не знаю. Скажу просто. Закон излучения Кирхгофа.
Отношение излучательной способности любого тела к его поглощательной способности одинаково для всех тел при данной температуре для данной частоты и не зависит от их формы и химической природы.

Поэтому если хочешь узнать какое тепловое излучение дает зеркало. Берешь тепловое излучения для абсолютно черного тела, и умножаешь на поглощательную способность зеркала (она мала).
Именно поэтому все термосы, чайники и т.д. делают зеркальными, а еду носят в фольге.
699 468400
Сколько всего возможных вариантов генотипа человека (мужчины/женщины) существует?
700 470289
Может ли появится органическая жизнь на планете, подобной нашей, только её постоянная температура в районе ~ -5 -10 градусов?
701 470514
>>63238

>А суперпозиция это же реальная вполне вещь и без наблюдателя,


>если бы её не было, никакой квантовый компьютер бы не смог считать.


Даже с наблюдателем квантовые компы могут считать, если уменьшить декогеренцию, и использовать квантовую алгоритмическую коррекцию ошибок
Цитата: https://ru.wikipedia.org/wiki/Шор,_Питер

>В 1995 году показал, что квантовые вычисления возможно проводить


>и при наличии не очень сильной декогеренции (необратимого воздействия внешней среды),


>если при этом использовать квантовую алгоритмическую коррекцию ошибок.

Тред утонул или удален.
Это копия, сохраненная 18 июня 2019 года.

Скачать тред: только с превью, с превью и прикрепленными файлами.
Второй вариант может долго скачиваться. Файлы будут только в живых или недавно утонувших тредах. Подробнее

Если вам полезен архив М.Двача, пожертвуйте на оплату сервера.
« /sci/В начало тредаВеб-версияНастройки
/a//b//mu//s//vg/Все доски